Интегралы что это: Интегралы для чайников – что это, как решать, примеры

Содержание

Интегралы для чайников – что это, как решать, примеры

За 4 минуты вы узнаете, что такое интегрирование. Как интеграл связан с производными. Чем отличается определенный интеграл от неопределенного. 5 примеров вычисления интегралов

Почему вы не знаете, как решать интегралы

А для чего нужны интегралы? Попробуйте сами себе ответить на этот вопрос.

Объясняя тему интегралов, учителя перечисляют малополезные школьным умам области применения. Среди них:

  • вычисление площади фигуры.
  • вычисление массы тела с неравномерной плотностью.
  • определение пройденного пути при движении с непостоянной скоростью.
  • и др.

Связать все эти процессы не всегда получается, поэтому многие ученики путаются, даже при наличии всех базовых знаний для понимания интеграла.

Главная причина незнания – отсутствие понимания практической значимости интегралов.

Нужна работа? Есть решение!

Более 70 000 экспертов: преподавателей и доцентов вузов готовы помочь вам в написании работы прямо сейчас.

Подробнее Гарантии Отзывы

Интеграл – что это?

Предпосылки. Потребность в интегрировании возникла в Древней Греции. В то время Архимед начал применять для нахождения площади окружности методы, похожие по сути на современные интегральные исчисления. Основным подходом для определения площади неровных фигур тогда был «Метод исчерпывания», который достаточно лёгок для понимания.

Суть метода. В данную фигуру вписывается монотонная последовательность других фигур, а затем вычисляется предел последовательности их площадей. Этот предел и принимался за площадь данной фигуры.

Метод исчерпывания для определения площади круга

В этом методе легко прослеживается идея интегрального исчисления, которая заключается в нахождении предела бесконечной суммы. В дальнейшем эта идея применялась учёными для решения прикладных задач астронавтики, экономики, механики и др.

Современный интеграл. Классическая теория интегрирования была сформулирована в общем виде Ньютоном и Лейбницем. Она опиралась на существовавшие тогда законы дифференциального исчисления. Для её понимания, необходимо иметь некоторые базовые знания, которые помогут математическим языком описать визуальные и интуитивные представления об интегралах.

Объясняем понятие «Интеграл»

Процесс нахождения производной называется дифференцированием, а нахождение первообразной – интегрированием.

Интеграл математическим языком – это первообразная функции (то, что было до производной) + константа «C».

Интеграл простыми словами – это площадь криволинейной фигуры. Неопределенный интеграл – вся площадь. Определенный интеграл – площадь в заданном участке.

Интеграл записывается так:

Каждая подынтегральная функция умножается на компонент «dx». Он показывает, по какой переменной осуществляется интегрирование. «dx» – это приращение аргумента. Вместо X может быть любой другой аргумент, например t (время).

Неопределённый интеграл

Неопределенный интеграл не имеет границ интегрирования.

Для решения неопределённых интегралов достаточно найти первообразную подынтегральной функции и прибавить к ней «C».

Пример решения неопределенного интеграла.

Определённый интеграл

В определенном интеграле на знаке интегрирования пишут ограничения «a» и «b». Они указаны на оси X в графике ниже.

Точки A и B на оси X – есть ограничение зоны определения интеграла

Для вычисления определенного интеграла необходимо найти первообразную, подставить в неё значения «a» и «b» и найти разность. В математике это называется формулой Ньютона-Лейбница:

Пример решения определенного интеграла

Таблица интегралов для студентов (основные формулы)

Скачайте формулы интегралов, они вам еще пригодятся

Как вычислять интеграл правильно

Существует несколько простейших операций для преобразования интегралов. Вот основные из них:

Вынесение константы из-под знака интеграла

Разложение интеграла суммы на сумму интегралов

Если поменять местами a и b, знак изменится

Можно разбить интеграл на промежутки следующим образом

Это простейшие свойства, на основе которых потом будут формулироваться более сложные теоремы и методы исчисления.

Примеры вычисления интегралов

Решение неопределенного интеграла

Решение определенного интеграла

Базовые понятия для понимания темы

Чтобы вы поняли суть интегрирования и не закрыли страницу от непонимания, мы объясним ряд базовых понятий. Что такое функция, производная, предел и первообразная.

Функция – правило, по которому все элементы из одного множества соотносятся со всеми элементами из другого.

Производная – функция, описывающая скорость изменения другой функции в каждой конкретной точке. Если говорить строгим языком, – это предел отношения приращения функции к приращению аргумента. Он вычисляется вручную, но проще использовать таблицу производных, в которой собрано большинство стандартных функций.

Приращение – количественное изменение функции при некотором изменении аргумента.

Предел – величина, к которой стремиться значение функции, при стремлении аргумента к определённому значению.

Пример предела: допустим при X равном 1, Y будет равно 2. Но что, если X не равен 1, а стремится к 1, то есть никогда её не достигает? В этом случае y никогда не достигнет 2, а будет только стремиться к этой величине. На математическом языке это записывается так: limY(X), при X –> 1 = 2. Читается: предел функции Y(X), при x стремящемся к 1, равен 2.

Как уже было сказано, производная – это функция, описывающая другую функцию. Изначальная функция может быть производной для какой-либо другой функции. Эта другая функция называется первообразной.

Заключение

Найти интегралы не трудно. Если вы не поняли, как это делать, прочитайте статью еще раз. Со второго раза становится понятнее. Запомните! Решение интегралов сводится к простым преобразованиям подынтегральной функции и поиска её в таблице интегралов.

Если текстовое объяснение вам не заходит, посмотрите видео о смысле интеграла и производной:

Интегралы – что это, как решать, примеры решений и объяснение для чайников обновлено: 16 апреля, 2020 автором: Научные Статьи.Ру

Интуитивное объяснение интеграла. Часть I — от умножения натуральных чисел до Ньютона и Лейбница

0. Предисловие

Математика представляет собой универсальный, мощный и элегантный раздел знания. По-сути её предмет и значение невозможно разделить с наиболее фундаментальными разделами философии — логикой, онтологией и теорией познания. Именно поэтому она касается прямо или косвенно всех аспектов любого прикладного или теоретического знания.

К сожалению, так сложилось, что многим (и мне) она, порой кажется, слишком сложной, недоступной, наукой для избранных. Между тем, так только кажется ! Безусловно, она требует интеллектуального напряжения, памяти, воображения и много чего ещё, как и многие другие интеллектуальные занятия.

Отличительными особенностями её являются:

  1. использование особой знаковой системы (цифры, буквы разных алфавитов, языковые правила и т. д.),

  2. логическая строгость (понятия, определения, суждения, правила вывода задаются в явном и точном виде),

  3. последовательность (не поймёшь пункт 3, если не понял пункты 1 и 2),

  4. высокая плотность информации на единицу текста (часто смысла в тексте гораздо больше, чем в текстах иного содержания).

Легко показать, что любой интеллектуально развитый человек регулярно использует те же мыслительные конструкции, что и математика. Когда мы говорим давайте рассмотрим десять каких-либо операций (алгоритм) вроде кулинарного рецепта или простейшей программы или рассмотрим какой-либо частный случай явления, определим его свойства, отношения с другими явлениями, изучим структуру — мы прибегаем к универсальным способам мышления, которые характерны для любого знания и в том числе математического.

Эта статья никогда бы не появилась на свет, если бы учебная литература была бы настолько совершенна, что могла бы легко объяснить, что такое интеграл. Перечитав десятки книг и статей я с уверенностью могу сказать, что ни одна из них не объясняет все нюансы этого вопроса так и таким образом, чтобы среднему, неискушённому человеку было всё абсолютно ясно.

Многие источники не удовлетворительны по следующим причинам:

  1. Говорят о какой-то площади под кривой при том, что читатель ни сном, ни духом не задумывался о площади, тем более под кривой и какой-то связи этой площади с универсальной идеей суммирования переменных величин

  2. Без интуитивного подведения читателя через сложение и умножение чисел, основательного разъяснения связи …. сразу бросаются к определению интеграла через предел римановской суммы

  3. Забывают рассказать об историческом процессе развития математики (зачем ввели интеграл, какие открытия этому предшествовали, что подвело к этому, как считали интегральные суммы до этого, как Ньютон и Лейбниц считали интегралы и т.д.)

  4. Не считают нужным или не хотят привести пару тройку простых примеров интегрирования из прикладных наук

  5. Сыпят доказательствами утверждений, которые новичку покажутся неуместными или второстепенными

  6. Забывают напомнить выводы, обозначения и утверждения, использованные или доказанные ранее

  7. Пропускают те или иные алгебраические преобразования, которые «очевидны» автору, но могут запутать новичка

Автору надоело чувствовать неясность и он решил взять дело в свои руки — расписать все аспекты так, чтобы было всё предельно ясно и понятно.

1. Предпосылки возникновения интегрирования

Интеграл и интегрирование являются неотъемлемыми и последовательными элементами исследования величин и функций. Интегрирование теснейшим образом связано с важнейшими способами анализа и исследования числовых функций — средними, предельными, бесконечно малыми, бесконечно большими величинами, пределами, дифференциалами, производными и т.д. А потому, без осознания и исследования этих понятий невозможно и формирование понятия интеграла.

Исторически и логически они развивались и развиваются слитно и нераздельно.

Во введении к книге «Развитие понятия интеграла» известный историк математики профессор Фёдор Андреевич Медведев так охарактеризовал сущность интегрирования и процесс его развития в науке «… Интегрирование представляет собой абстрактное выражение разнообразнейших способов измерения величин, и по мере вовлечения в человеческое познание всё новых и новых объектов реальной действительности математики создают всё более и более общие схемы интеграционных процессов с тем, чтобы охватить всё расширяющийся круг объектов, подлежащих измерению» [1].

Как известно осознание самостоятельной значимости и полноценное развитие математики начались в Древней Греции. Постепенное накопление прикладных знаний о различного рода вычислительных, логических и геометрических задачах неизбежно привело к формированию теоретических начал и абстрактных представлений о существе многих математических идей.

Корпус прикладных и теоретических знаний накапливался и формировался шаг за шагом за счёт осмысления логического устройства мышления, применения арифметических операций, составления и решения алгебраических уравнений, построения и изучения свойств плоских и объёмных геометрических фигур.

2. Геометрический и аналитико-алгебраический смысл интегрирования

Естественным образом, возникает два вида задач, которые отражают два смысла интегрирования: — геометрический и аналитико-алгебраический. Первый – отыскание площади плоской фигуры под произвольной кривой (квадратура) и отыскание объёма (кубатура). Второй – подсчёт суммарного значения некой переменной величины [2], которая изменяется, принимает различные значения сообразно единицам времени, длины и т. д.

Согласно дошедшим до нас источникам, именно отыскание квадратуры является первой формой постановки задачи интегрирования. Задача явно сформулирована и решена в трудах Евдокса Книдского (сформулировал метод исчерпывания, позднее развитый в XVI веке в метод неделимых), Евклида и Архимеда. Древнегреческих математиков интересовали задачи отыскания площади круга, поверхности сферы, сегмента параболы, а также объёма шара, цилиндра, пирамиды, конуса, тетраэдра и ряда других геометрических фигур.

Под проведением квадратуры понималось построение с помощью циркуля и линейки квадрата, равновеликого заданной фигуре (то есть имеющего такую же площадь) или прямое вычисление соответствующей площади. Вероятно связи геометрии и анализа если и обнаруживались, то интуитивно и неявно. Во всяком случае координатный метод и понятия дифференциального исчисления точно не были известны, хотя и почти что точно были так или иначе интуитивно восприняты и неявно затронуты.

Что касается второго типа задач. Интегралы часто описываются как площадь под кривой. Это описание сбивает с толку. Точно также, как если сказать, что умножение — это нахождение площади прямоугольника. Именно понимание сущности умножения применительно к различного рода частным случаям позволяет понять аналитико-алгебраическую суть интегрирования.

Понимание и использование простейших случаев умножения, к примеру, умножения натуральных чисел, было известно с древнейших времён.

Однако, за всеми частными случаями умножения находится определённая общность. Вот как можно описать умножение чисел из различных числовых множеств:

  • В случае с натуральными числами. К примеру, умножим число 3 на число 4, то есть 3 × 4. Умножение — это повторяющееся сложение, то есть произведение чисел получим сложив число три четыре раза или наоборот сложив число четыре три раза [3].

  • В случае с вещественными числами.

    • Возьмём одно рациональное число — дробь, а другое целое. К примеру, умножим 3,5 на 2, то есть — 3,5 × 2. Умножение — это повторяющееся сложение, произведение получим сложив число три целых и пять десятых два раза. Также, получить произведение можно путём сложения произведений вначале целой части числа 3,5 то есть 3 на 2, а затем дробной то есть 0,5 на 2. Для целой части — сложим число три два раза, а для дробной части — возьмём единицу разделим на десять, затем возьмём пять частей от деления то есть пять десятых и сложим два раза.

    • Возьмём два рациональных числа — две дроби и получим произведение. К примеру, умножим 3,5 на 2,1 то есть — 3,5 × 2,1, произведение получим сложив произведение 3,5 на 2 и 3,5 на 0,1 [4]. Словесно это будет выглядеть следующим образом, для первого произведения — сложим число три целых пять десятых два раза, для второго — разделим число три целых пять десятых на десять частей и возьмём одну часть то есть одну десятую.

    • В случае с отрицательными числами (-2,3 × 4,3), умножение — сумма произведений и разворот числовой оси или иными словами отражение суммарного значения произведения — в данном случае числа 9,89 относительно начала отсчёта, то есть числа ноль, в результате получаем -9,89.

  • В случае с комплексными числами (3 × 3i), умножение выступает вращением и масштабированием.

Мы ходим вокруг да около «применения» одного числа к другому, и действия, которые мы применяем (повторное суммирование, масштабирование, зеркальное отображение или вращение), могут быть разными. Интегрирование — это всего лишь еще один шаг в этом направлении.

Когда мы умножаем числа мы повторяем сложение, где в каждом слагаемом знаем какие находятся операнды, а именно — повторяющиеся числа.

К примеру, если мы хотим вычислить пройденный путь телом, движущимся с одинаковой скоростью в каждый момент времени, то мы просто перемножим скорость на время (значение функции скорости одинаково, а геометрически грубо говоря одинаково во всем прямоугольнике).

Но изменяющаяся скорость требует совмещения скорости и времени по частям (момент за моментом, секунда за секундой). В каждый момент скорость может быть разной.

Вот как это выглядит в большой перспективе:

  • Обычное умножение (прямоугольник): берем расстояние, на которое мы продвинулись за секунду, предполагая, что эта величина была постоянной во все последующие секунды движения, и «масштабируем ее».

  • Интегрирование (по частям): рассматриваем время как ряд мгновений, в каждое из которых скорость разная. Суммируем расстояния, пройденные в каждое из мгновений (секунд, миллисекунд и т. д.).

То есть, интегральную сумму (значение интеграла, определённый интеграл) можно определить, как максимально точную сумму значений искомой переменной величины

при её изменении в промежутке от до где а .

Точность достигается в пределе, то есть при всё большем уменьшении размера промежутков между значениями или, что тоже самое, при всё большом увеличении числа отрезков (числа — обозначающего индекс-номер последнего отрезка)

Несомненно греческих и более поздних мыслителей интересовали задачи на отыскание суммарного значения переменных величин. Вероятно их устраивало простое суммирование значений переменной величины, приближённые вычисления. Если мы возьмём приращение переменной равное единице, то интеграл приближённо будет равен сумме значений функции в рассматриваемом промежутке.

В дальнейшем, начиная с XVI века (работы Галилея, Кеплера, Кавальери и других о методе неделимых) понимание интегрирования постепенно совершенствовалось и развивалось пока не достигло формализации у Бернхарда Римана в середине XIX века и дальнейшего обобщения.

3. Интуитивные способы отыскания значения интеграла

Итак, каким же образом вычислить интегральную сумму ? Можно попробовать несколько способов:

  1. Умножить совокупное приращение переменной на значение функции и получить площадь прямоугольника, который добавит значительный излишек, либо срежет значительную часть в зависимости от того какое значение функции мы выберем. Вручную мы можем подобрать такое значение функции, что при умножении её на приращение переменной мы получим довольно точное значение площади (определённого интеграла в промежутке). Для этого нам потребуется провести линию так, чтобы площадь излишка примерно равнялась срезанной площади. Однако, это не даст нам универсального метода отыскания значения искомой величины.

2. Сложить произведения приращения переменной на значение функции в соответствующих точках, получив тем самым сумму площадей прямоугольников, внешне напоминающих лестницу (ступеньки). В самом простом случае приращение равно единице. На этом методе и основано формальное определение определённого интеграла, данное Б. Риманом. О нём мы поговорим ниже.

3. Воспользоваться иными так называемыми численными способами отыскания значения интегральной суммы (интеграла).

4. Отыскание значения интеграла через отыскание первообразной

Однако есть более изящный и универсальный способ вычисления интегральной суммы, который был открыт Исааком Ньютоном и Готфридом Лейбницом. Этот способ устанавливает фундаментальную связь дифференцирования (производной) и интегрирования (первообразной).

Чтобы рассмотреть суть открытия, необходимо последовательно прийти к ряду идей и рассуждений.

Пусть имеется некоторая функция от числовой переменной — Обозначим её [5].

Следует отметить несколько обстоятельств относительно рассматриваемой функции:

Для наглядности изобразим график рассматриваемой функции в виде произвольной кривой.

Пусть мы хотим отыскать всю или часть совокупного значения (аналитико-алгебраический смысл интегрирования) или площадь под кривой (геометрический смысл). Выберем промежуток между двумя точками и и продолжим наши рассуждения.

Искомое значение представляет собой функцию и очевидно, что оно будет зависеть от размера промежутка и того значения изначальной функции, которое она принимает в каждой точке этого промежутка. Также, очевидно, что промежуток значений переменной для изначальной функции и функции площади будет одинаковым [6].

Сказанное выше легко показать и увидеть на графике.

Заметим, что значения функции площади не равны значению изначальной функции при том же значении переменной [7]. Значения площади постоянно возрастает слева-направо, то есть при каждом шаге приращения промежутка суммирования (интегрирования).

Пусть теперь исследуемая функция является функцией скорости движения материальной точки (тела) по некоторой траектории. Тогда, очевидно, по определению производной, что скорость в конкретный момент времени — это первая производная пути (координаты) по времени

Если скорость это производная пути и мы знаем аналитическое выражение её выражающее, то мы можем найти выражение для самого пути то есть для самой функции. Мы можем это сделать через операцию, обратную нахождению производной то есть через отыскание первообразной. Это справедливо, поскольку производная и соответствующее ей семейство первообразных единственны.

Данный вывод можно обобщить на все интегрируемые функции.

Далее, легко понять из простых арифметических и геометрических соображений, что значение интегральной суммы (площади) будет равно разности значений полученной функции (первообразной), взятых в соответствующих точках [8].

То есть если требуется найти интегральную сумму в промежутке от до , где первое и второе — некоторые произвольные значения переменной, то необходимо вычислить разность

Указанная сумма и есть определённый интеграл, который записывается, как

[1]. Медведев Ф.А. Развитие понятия интеграла. — М.: Наука, 1974. С. 4

[2]. Имеется ввиду сумма значений переменной, которая является элементом интегрирования, интегрируемой величиной.

[3]. Не имеет значения каким образом будем вычислять произведение, так как от перестановки множителей произведение не меняется, то есть данная операция обладает свойством коммутативности.

[4]. 3,5 · 2 + 3,5 · 0,1 = 3,5 (2 + 0,1) = 3,5 · 2,1.

[5]. Вместоможет быть любое обозначение, к примеру, — это не имеет значения. Буквавсего лишь обозначает имя для функции, а скобки отделяют имя от сущностей — обычно числовых переменных над которыми совершаются те или иные операции, дающие в результате значение функции.

[6]. Переменная-аргумент — одна и таже, то есть иными словами значения переменной-аргумента в точках для и одно и тоже. Далее, мы покажем, что производная , то есть можно записать или .

[7]. То есть . К примеру, пусть функция задана выражением . Тогда, при , , а значение . Если. Тогда, при , , а значение .

[8]. Пусть имеется точка, число 7 и 10, чтобы найти величину промежутка между этими значениями надо найти разность то есть 10 — 7 = 3.

ИНТЕГРАЛ – это… Что такое ИНТЕГРАЛ?

  • ИНТЕГРАЛ — (обозначение т ). Математический символ, используемый в ИСЧИСЛЕНИИ, представляющий операцию суммирования. Интеграл функции f(x), записанный как т f(x)dx, может представлять площадь фигуры, ограниченной кривой y=f(x) и осью абсцисс.

    ИНТЕГРИРОВАНИЕ …   Научно-технический энциклопедический словарь

  • интеграл — а, м. intégrale f. <лат. integer целый. Математическое понятие о целой величине как сумме своих бесконечно малых частей. Нахождение интеграла. БАС 1. Найти интеграл уравнения. 1766. Котельников Геодет 175. // Сл. 18. Алферинька недурно… …   Исторический словарь галлицизмов русского языка

  • ИНТЕГРАЛ — муж., мат., лат. конечная, измеримая величина, в отношении к бесконечно малой части ее, к дифференциалу. Интегральное вычисление, искусство отыскивать интеграл по дифференциалу. Интегрировать, вычислять, находить интеграл; интеграция жен.… …   Толковый словарь Даля

  • ИНТЕГРАЛ — (вово лат., от лат. integer ценный). В математике количество, дифференциал которого равен данной величине. Словарь иностранных слов, вошедших в состав русского языка. Чудинов А.Н., 1910. интеграл (лат. integer целый) лет. 1) неопределенный и. от… …   Словарь иностранных слов русского языка

  • интеграл — первообразная, термин Словарь русских синонимов. интеграл сущ., кол во синонимов: 2 • первообразная (1) • …   Словарь синонимов

  • интеграл — — [Я.Н.Лугинский, М.С.Фези Жилинская, Ю.С.Кабиров. Англо русский словарь по электротехнике и электроэнергетике, Москва, 1999 г.] интеграл Есть два различных понятия — неопределенный И. и определенный И. Говорят, что функция f(x) имеет …   Справочник технического переводчика

  • ИНТЕГРАЛ — (от латинского integer целый), одно из основных понятий интегрального исчисления …   Современная энциклопедия

  • ИНТЕГРАЛ — (от лат. integer целый) см. Интегральное исчисление …   Большой Энциклопедический словарь

  • ИНТЕГРАЛ — ИНТЕГРАЛ, интеграла, муж. (от лат. integer целый) (мат.). Конечная измеримая величина в отношении к бесконечно малой части ее к диференциалу.

    Толковый словарь Ушакова. Д.Н. Ушаков. 1935 1940 …   Толковый словарь Ушакова

  • ИНТЕГРАЛ — [тэ ], а, муж. В математике: величина, получающаяся в результате действия, обратного дифференцированию. | прил. интегральный, ая, ое. Интегральное исчисление. Толковый словарь Ожегова. С.И. Ожегов, Н.Ю. Шведова. 1949 1992 …   Толковый словарь Ожегова

  • Определённый интеграл и методы его вычисления

    В каждой главе будут и задачи для самостоятельного решения, к которым можно посмотреть ответы.

    Определённым интегралом от непрерывной функции f(x) на конечном отрезке [a, b] (где ) называется приращение какой-нибудь её первообразной на этом отрезке. (Вообще, понимание заметно облегчится, если повторить тему неопределённого интеграла) При этом употребляется запись

    Как видно на графиках внизу (приращение первообразной функции обозначено ), определённый интеграл может быть как положительным, так и отрицательным числом (Вычисляется как разность между значением первообразной в верхнем пределе и её же значением в нижнем пределе, т. е. как F(b) – F(a)).

    Числа a и b называются соответственно нижним и верхним пределами интегрирования, а отрезок [a, b] – отрезком интегрирования.

    Таким образом, если F(x) – какая-нибудь первообразная функция для f(x), то, согласно определению,

                (38)

    Равенство (38) называется формулой Ньютона-Лейбница. Разность F(b) – F(a) кратко записывают так:

    Поэтому формулу Ньютона-Лейбница будем записывать и так:

                       (39)

    Докажем, что определённый интеграл не зависит от того, какая первообразная подынтегральной функции взята при его вычислении. Пусть F(x) и Ф(х) – произвольные первообразные подынтегральной функции. Так как это первообразные одной и той же функции, то они отличаются на постоянное слагаемое: Ф(х) = F(x) + C. Поэтому

    Тем самым установлено, что на отрезке [a, b] приращения всех первообразных функции f(x) совпадают.

    Таким образом, для вычисления определённого интеграла необходимо найти любую первообразную подынтегральной функции, т.е. сначала следует найти неопределённый интеграл. Постоянная С из последующих вычислений исключается. Затем применяется формула Ньютона-Лейбница: в первообразную функцию подставляется значение верхнего предела b, далее – значение нижнего предела a и вычисляется разность F(b) – F(a). Полученное число и будет определённым интегралом..

    При a = b по определению принимается

    Пример 1. Вычислить определённый интеграл

    Решение. Сначала найдём неопределённый интеграл:

    Применяя формулу Ньютона-Лейбница к первообразной

    (при С = 0), получим

    Однако при вычислении определённого интеграла лучше не находить отдельно первообразную, а сразу записывать интеграл в виде (39).

    Пример 2. Вычислить определённый интеграл

    Решение. Используя формулу

    получим

    Найти определённый интеграл самостоятельно, а затем посмотреть решение

    Теорема 1. Определённый интеграл с одинаковыми пределами интегрирования равен нулю, т.е.

    Это свойство содержится в самом определении определённого интеграла. Однако его можно получить и по формуле Ньютона-Лейбница:


    Теорема 2. Величина определённого интеграла не зависит от обозначения переменной интегрирования, т.е.

                             (40)

    Пусть F(x) – первообразная для f(x). Для f(t) первообразной служит та же функция F(t), в которой лишь иначе обозначена независимая переменная. Следовательно,

    На основании формулы (39) последнее равенство означает равенство интегралов

    и


    Теорема 3. Постоянный множитель можно выносить за знак определённого интеграла, т.е.

                        (41)       


    Теорема 4. Определённый интеграл от алгебраической суммы конечного числа функций равен алгебраической сумме определённых интегралов от этих функций, т. е.

                (42)


    Теорема 5. Если отрезок интегрирования разбит на части, то определённый интеграл по всему отрезку равен сумме определённых интегралов по его частям, т.е. если

    то

                      (43)


    Теорема 6. При перестановке пределов интегрирования абсолютная величина определённого интеграла не меняется, а изменяется лишь его знак, т.е.

                     (44)


    Теорема 7 (теорема о среднем). Определённый интеграл равен произведению длины отрезка интегрирования на значение подынтегральной функции в некоторой точке внутри его, т.е.

       (45)


    Теорема 8. Если верхний предел интегрирования больше нижнего и подынтегральная функция неотрицательна (положительна), то и определённый интеграл неотрицателен (положителен), т. е. если



    Теорема 9. Если верхний предел интегрирования больше нижнего и функции и непрерывны, то неравенство

    можно почленно интегрировать, т.е.

                 (46)


    Свойства определённого интеграла позволяют упрощать непосредственное вычисление интегралов.

    Пример 5. Вычислить определённый интеграл

    Используя теоремы 4 и 3, а при нахождении первообразных – табличные интегралы (7) и (6), получим


    Пусть f(x) – непрерывная на отрезке [a, b] функция, а F(x) – её первообразная. Рассмотрим определённый интеграл

                    (47)

    где

    ,

    а через t  обозначена переменная интегрирования, чтобы не путать её с верхней границей. При изменении х меняется и опредёленный интеграл (47), т.е. он является функцией верхнего предела интегрирования х, которую обозначим через Ф(х), т.е.

                           (48)

    Докажем, что функция Ф(х) является первообразной для f(x) = f(t). Действительно, дифференцируя Ф(х), получим

    так как F(x) – первообразная для f(x), а F(a) – постояная величина.

    Функция Ф(х) – одна из бесконечного множества первообразных для f(x), а именно та, которая при x = aобращается в нуль. Это утверждение получается, если в равенстве (48) положить x = aи воспользоваться теоремой 1 предыдущего параграфа.

    При выводе формулы интегрирования по частям было получено равенство u dv = d (uv) – v du. Проинтегрировав его в пределах от a до b и учитывая теорему 4 параграфа этой статьи о свойствах определённого интеграла, получим

    Как это следует из теоремы 2 параграфа о свойствах неопределённого интеграла, первый член в правой части равен разности значений произведения uv при верхнем и нижнем пределах интегрирования. Записав эту разность кратко в виде

     

    получаем формулу интегрирования по частям для вычисления определенного интеграла:

               (49)

    Пример 6. Вычислить определённый интеграл

    Решение. Интегрируем по частям, полагая u = ln x, dv = dx; тогда du = (1/x)dx, v = x. По формуле (49) находим

    Найти определённый интеграл по частям самостоятельно, а затем посмотреть решение

    Перейдём к вычислению определённого интеграла методом замены переменной. Пусть

    где, по определению, F(x) – первообразная для f(x). Если в подынтегральном выражении произвести замену переменной

    то в соответствии с формулой (16) можно записать

    В этом выражении

    первообразная функция для

    В самом деле, её производная, согласно правилу дифференцирования сложной функции, равна

    Пусть α и β – значения переменной t , при которых функция

    принимает соответственно значения aи b, т.е.

    Тогда

    Но, согласно формуле Ньютона-Лейбница, разность F(b) – F(a) есть

    поскольку F(x) – первообразная для f(x).

    Итак,

               (50)

    Это и есть формула перехода к новой переменной под знаком определённого интеграла. С её помощью определённый интеграл

    после замены переменной

    преобразуется в определённый интеграл относительно новой переменной t. При этом старые пределы интегрирования a и b заменяются новыми пределами и . Чтобы найти новые пределы, нужно в уравнение

    поставить значения x = aи x = b, т.е. решить уравнения

    и

    относительно и . После нахождения новых пределов интегрирования вычисление определённого интеграла сводится к применению формулы Ньютона-Лейбница к интегралу от новой переменной t. В первообразной функции, которая получается в результате нахождения интеграла, возвращаться к старой переменной нет необходимости.

    При вычислении определённого интеграла методом замены переменной часто бывает удобно выражать не старую переменную как функцию новой, а, наоборот, новую – как функцию старой.

    Пример 9. Вычислить определённый интеграл

    Решение. Произведём замену переменной, полагая

    Тогда dt = 2x dx, откуда x dx = (1/2) dt, и подынтегральное выражение преобразуется так:

    Найдём новые пределы интегрирования. Подстановка значений x = 4 и x = 5 в уравнение

    даёт

    а

    Используя теперь формулу (50), получим

    После замены переменной мы не возвращались к старой переменной, а применили формулу Ньютона-Лейбница к полученной первообразной.

    Найти определённый интеграл заменой переменной самостоятельно, а затем посмотреть решение

    Начало темы “Интеграл”

    Продолжение темы “Интеграл”

    Поделиться с друзьями

    📝Что такое интеграл?

    В переводе с латинского языка интеграл означает «целый». Это одно из наиболее важных и распространенных понятий в высшей математике, которое появилось из-за необходимости находить функции по их производным или измерять объёмы, площади, работу нескольких сил за конкретный промежуток времени, длины дуг и т.д. В соответствии с этими задачами принято выделять определённые и неопределенные интегралы.

    Содержание статьи:

    Обозначение

    Первым символ для обозначения интегрирования придумал Ньютон. Он применял для этого небольшой квадрат. Однако данное обозначение не получило серьезного распространения. Сегодняшнее обозначение неопределенного интеграла было придумано в 1675 году Лейбницем:

    Что касается обозначения определённого интеграла, где указаны пределы интегрирования, то его в 1819 году предложил Жан Батист Фурье.

    Виды интегралов

    Первообразная функции f(x) — функция F(x), производная которой при любом значении х равняется f(x). Добавляя постоянную к первообразной определенной функции, снова можно получить первообразную этой же функции. Соответственно, имея единственную первообразную F(x) функции f(x), можно получить единое выражение всех первообразных данной функции в виде F(x) + С. Подобное выражение первообразных принято называть неопределённым интегралом функции f(x):

    Одно из главных правил интегрального исчисления определяет, что любая непрерывная функция f(x) имеет неопределённый интеграл.

    Что касается определённого интеграл от функции f(x) с верхним пределом b и нижним пределом а, то он определяется в качестве разности:

    где F(x) является первообразной функции f(x).

    Определённый интеграл можно выразить посредством любой первообразной F(x). Верным является и обратное. Первообразную F(x) можно записать в следующем виде:

    В этой формуле а – это произвольная константа. Таким образом, интеграл можно записать в виде:

    История возникновения интеграла

    Если углубиться в историю, то можно утверждать, что интегрирование зародилось в древнем Египте, приблизительно в 1800 году до нашей эры. Первой известной методикой вычисления интегралов считается способ исчерпывания Евдокса. Он предпринимал попытки найти объёмы и площади фигур, разрывая их на несколько частей, для которых уже известны площадь или объём. Через некоторое время данная методика была развита Архимедом. Он применял ее для вычислений площадей парабол и примерного расчёта площади круга. Подобные методы независимо разрабатывались в Китае в 3 столетии нашей эры Лю Хуэйем. Он использовал их с целью определения площади круга.

    Следующий внушительный прогресс в исчислении интегралов произошел только в XVI веке. В работах с методом неделимых Кавальери, а также в научных трудах Ферма, были заложены основы сегодняшнего интегрального исчисления.

    Последующие шаги были сделаны в середине XVII столетия Торричелли и Барроу, которые предоставили первые намеки на взаимосвязь между дифференцированием и интегрированием.

    Зачем и кому нужны интегралы?

    Ученые стремятся любые физические явления выражать в виде математических формул. Когда в руках есть определенная формула, то в дальнейшем уже можно с ее помощью посчитать все, что необходимо. А интеграл является одним из главных инструментов работы с любыми функциями.

    К примеру, имея формулу круга, можно посредством интеграла вычислить его площадь. Если есть формула шара, то можно вычислить его объем. Посредством интегрирования можно найти работу, энергию, массу, давление, электрический заряд и прочие важные величины.

    Материалы по теме:

    Поделиться с друзьями:

    Загрузка…

    определение, история развития, применение интегралов на практике

    Содержание:

    Имеется несколько типов интегралов: неопределенный и определенный интегралы, интеграл Римана и Римана-Стилтьеса, интеграл Лебега и Лебега-Стилтьеса, интеграл Даниэля. По области интегрирования интегралы подразделяются на кратные, криволинейные и поверхностные интегралы.

    Историческая справка

    Интегрирование берет свое начало ещё в древнем Египте примерно с 1800 года до н. э., о чем свидетельствует Московский математический папирус (или математический папирус Голенищева). Первым известным методом для расчёта интегралов является метод для исследования площади или объёма криволинейных фигур – метод исчерпывания Евдокса (Евдокс Книдский (ок. 408 г. до н.э. – ок. 355 г. до н.э.) – древнегреческий математик, механик и астроном), который был предложен примерно в 370 до н. э. Суть этого метода заключается в следующем: фигура, площадь или объем которой пытались найти, разбивалась на бесконечное множество частей, для которых площадь или объём уже известны. Этот метод получил свое дальнейшее развитие в работах древнегреческого математика, физика и инженера Архимеда (287 до н.э. – 212 до н.э.) для расчёта площадей парабол и приближенного расчёта площади круга. {a} \sqrt{x} d x$

    Используя математическую индукцию, он смог обобщить свои результаты для интегралов от многочленов до четвёртой степени. Таким образом, он был близок к поиску общей формулы для интегралов от полиномов не выше четвёртой степени.

    Следующий значительный толчок в исчислении интегралов состоялся лишь в 16 веке в работах итальянского математика Бонавентура Франческо Кавальери (1598 – 1647), в которых описывался предложенный им метод неделимых, а также в работах французского математика Пьера де Ферма (1601 – 1665). Этими учеными были заложены основы современного интегрального исчисления. Дальнейшее развитие связано с деятельностью английского математика, физика и богослова Исаака Барроу (1630 – 1677) и итальянского математика и физика, ученика Галилея Эванджелиста Торричелли (1608 – 1647), которые представили первые намеки на связь между интегрированием и дифференцированием.

    За время становления интегрального исчисления менялось и обозначение интеграла. Английский физик, механик, математик и астроном Исаак Ньютон (1643 – 1727) использовал, правда не во всех своих работах, в качестве символа интегрирования значок квадрата перед обозначением функции или вокруг него, а также вертикальную черту над функцией, но эти обозначения не получили широкого распространения. Современное обозначение неопределённого интеграла было введено немецким философом, логиком, математиком, механиком, физиком, юристом, историком, дипломатом, изобретателем и языковедом Готфридом Вильгельмом Лейбницем (1646 – 1716) в 1675 году. Он образовал символ интеграла из буквы “длинная s” (от первой буквы слова Summa – сумма) Современное обозначение определённого интеграла, с указанием пределов интегрирования, было впервые предложено французским математиком и физиком Жаном Батистом Жозефом Фурье (1768 – 1830) в 1819-20 годах. Сам термин “интеграл” придумал швейцарский математик Якоб Бернулли (1654 – 1705) в 1690 году.

    Применение интегралов на практике

    Основной задачей дифференциального исчисления является определение для заданной функции $F(x)$ ее производной $F^{\prime}(x)=f(x)$ или ее дифференциала $F^{\prime}(x) d x=f(x) d x$ . Обратная задача, состоящая в определении функции $F(x)$ по ее известным производной $f(x)$ или дифференциалу $f(x) d x$, представляет собой основную задачу интегрального исчисления.

    Читать дальше: неопределенный интеграл и понятие первообразной.

    Обзор методов вычисления интегралов по времени и пространству

    Интегрирование — один из важнейших математических инструментов, особенно в численном моделировании. Например, дифференциальные уравнения в частных производных обычно выводятся из интегральных уравнений сохранения. Когда возникает необходимость численного решения уравнения в частных производных, интегрирование также играет важную роль. В этой статье приведен обзор методов и подходов интегрирования, доступных в COMSOL Multiphysics, а также конкретные примеры их использования.

    Важность интегралов

    В COMSOL используется метод конечных элементов, который преобразует описывающее некоторый процесс уравнение в частных производных в интегральное уравнение — другими словами, в слабую форму (weak form). {t_1}\int_{\Omega}F(u)\ \mathrm{d A} \mathrm{d} t

    где [t_0,t_1] — это временной интервал, \Omega — это пространственная область, а F(u) — это произвольное выражение, включающее зависимую переменную u и произвольные функции от нее, в том числе производные по пространству, времени, а также любой другой величине.

    Наиболее удобный способ вычисления интегралов — использование узла Derived Values (Расчет выражений) в разделе Results (Результаты) ленты Ribbon или дерева модели (Лента Ribbon отсутствует в том случае, если ваш компьютер работает не под управлением ОС Windows®).

    Добавление операций расчета пространственных интегралов по объему, поверхности или линии в узле Derived Values (Расчет выражений)

    Вы можете обратиться к любому доступному решению, выбрав соответствующий набор данных (data set). В поле Expression (Выражение) вводится подынтегральная функция, включающая зависимые или производные переменные. Для данных расчета во временной области пространственный интеграл вычисляется на каждом временном шаге. В качестве альтернативы, в окне Settings (Настройки) узла Data Series Operations (Операции с массивами данных) можно выбрать опцию Integral (Интегрирование), что позволит вычислить общий пространственно-временной интеграл.

    Пример настроек вычисления интегралов по поверхности (Surface Integration) с дополнительным вычислением интеграла по времени в разделе Data Series Operations.

    Оператор Average (Усреднение) — еще одна операция в разделе Derived Values, связанная с вычислением интегралов. Оператор вычисляет интеграл и делит его на объем, площадь или длину выбранной области. Операция Averageв узле Data Series Operations аналогично вводит деление на продолжительность временного диапазона. Операторы узла Derived Values — важный инструмент, однако их можно использовать только во время постобработки, а значит с их помощью можно рассчитать далеко не любой интеграл. Именно поэтому в COMSOL представлены другие более мощные и гибкие инструменты для вычисления интегралов.2. Стационарное и нестационарное решение (в момент времени 100 секунд) представлены на иллюстрациях ниже.


    Стационарное решение, нажмите на изображение для увеличения.
    Нестационарное решение (для момента времени 100 секунд), нажмите на изображение для увеличения.

    Вычисление пространственного интеграла с использованием операторов узла Component Coupling

    Операторы узла Component Coupling (Сопряжение компонентов) используются в тех случаях, когда, например, в одном выражении объединяются несколько интегралов, или интегралы требуются в процессе вычислений, или требуется множество контурных интегралов. Операторы данного узла определяются в разделе Definitions (Определения). На этом этапе режультат использования оператора не просчитывается, а указываются только их название и выборки областей.


    Добавление операторов через узел Component Couplings

    В нашем примере мы для начала хотим вычислить пространственный интеграл для стационарного распределения температуры, равный

    \int_{\Omega}T(x,y)\ \mathrm{d}x\mathrm{d}y = 301.65

    В пакете COMSOL оператор вычисления интеграла по умолчанию получает имя intop1.


    Окно настроек оператора интегрирования.
    Расчет результата интегрирования через оператор.

    Теперь давайте рассмотрим, как оператор интегрирования может использоваться непосредственно в процессе расчета модели. С его помощью мы могли бы, например, выяснить, какая нагревательная мощность потребуется для получения средней температуры 303.15 К, то есть температуры, на 10 К превышающей температуру окружающей среды. Прежде всего нам необходимо вычислить разницу между требуемым и действительным средними значениями. Среднее значение вычисляется путем деления интеграла от T на интеграл от постоянной функции 1, который равен площади области. Нетрудно догадаться, что вычисление подобного вида легко выполнить с помощью представленного в COMSOL оператора Average (Усреднение), см. комментарии выше. По умолчанию данный оператор получает название aveop1.2. Т.е. полученное значение можно задать в качестве граничного условия для общего входящего теплового потока, чтобы средняя температура в рассматриваемой области стала равна 303.15 К.

    Вычисление неопределенного интеграла посредством оператора интегрирования

    В своих обращениях в службу поддержки пользователи часто задают один и тот же вопрос: как рассчитать неопределенный пространственный интеграл? Для этой цели нам также пригодится оператор интегрирования, задаваемый через Component Couplings. Нахождение неопределенного интеграла — операция, обратная дифференцированию. Неопределенный интеграл позволяет вычислять площади произвольных областей, ограниченных графиками функций. Одна из самых важных прикладных задач — вычисление вероятностей в статистическом анализе. Для того чтобы это продемонстрировать, мы зафиксируем y=0 и обозначим неопределенный интеграл от T(x,0) как u(x). Это значит, что \frac{\partial u}{\partial x}=T(x,0). Тогда неопределенный интеграл имеет вид

    u(\bar x) = \int_0^{\bar x}T(x,0)\mathrm{d} x

    Здесь мы используем \bar x, чтобы отличать переменную интегрирования от внешней переменной.1T(x,0)\cdot(x\leq\bar x)\ \mathrm{d} x

    Во-вторых, нам понадобится оператор вычисления интеграла, который будет действовать на нижней границе области из примера. Давайте обозначим его как intop2. В-третьих, мы должны отличать переменную интегрирования от внешней переменной. Принятые обозначения для такого случая: x называется источником (source), а \bar x — точкой назначения (destination). При использовании операторов интегрирования доступен встроенный оператор dest, который позволяет явно оглашать, что соответствующее выражение не относится к переменным интегрирования. Точнее, это значит, что в COMSOL \bar x=dest(x). Объединив логическое выражение с оператором dest, мы получим выражение вида T*(x<=dest(x)), которое является именно тем входным выражением, которое требуется для intop2. Объединив все вместе, мы можем вычислить неопределенный интеграл, воспользовавшись выражением intop2(T*(x<=dest(x))). Результат данной операции можно проиллюстрировать следующим графиком:


    Как построить график неопределенного интеграла с помощью оператора интегрирования, оператора dest и логического выражения.

    В пакете COMSOL дополнительно доступны еще два оператора вычисления интеграла, а именно общая проекция (general projection) и линейная проекция (linear projection). Эти операторы можно использовать для получения множества контурных интегралов в любом направлении в области. Другими словами, вычисление интеграла производится только вдоль одного измерения. В результате мы получаем функцию размерности на единицу меньше, чем размерность области. Для двухмерного примера результатом будет одномерная функция, которая может быть рассчитана на любой границе. Более подробная информация об использовании данных операторов будет представлена в одной из следующих публикаций в нашем компоративном блоге.

    Вычисление пространственного интеграла посредством дополнительного физического интерфейса

    Наиболее гибким способом вычисления пространственных интегралов является техника с добавлением дополнительного PDE-интерфейса. Давайте вспомним пример выше с неопределенным интегралом и предположим, что мы хотим вычислить неопределенный интеграл не только для y=0. Данная задача может быть сформулирована в виде дифференциального уравнения в частных производных

    \frac{\partial u}{\partial x}=T(x,y)

    с граничным условием типа Дирихле u=0 на левой границе. Расчет такого уравнения проще всего реализовать в физическом (математическом) интерфейсе Coefficient Form PDE (Дифференциальное уравнение в частных производных, коэффициентная форма записи), который потребует следующих настроек:


    Вычисление пространственного интеграла посредством дополнительного PDE-интерфейса.

    Зависимая переменная u представляет собой неопределенный интеграл по x и доступна в процессе расчета модели и в постобработке. Помимо гибкости, дополнительным преимуществом данного подхода является точность, так как интеграл рассчитывается не вспомогательными инструментами на основе уже определенного распределения переменной, а непосредственно в процессе расчета с учетом алгоритмов оценки погрешностей и т.{100}T(x,y,t)\ \mathrm{d} t

    На поверхностном графике ниже представлен результирующий интеграл, являющийся функцией пространственных переменных (x,y):


    Использование оператора timeavg – оператора вычисления интеграла по времени.

    Схожие операторы существуют для вычисления интегралов на сферических зонах, а именно ballint, circint, diskint и sphint.

    Вычисление временного интеграла посредством дополнительного физического интерфейса

    В случае если временные интегралы нужно использовать непосредственно в модели в процессе расчета, вам будет необходимо задать их как дополнительные зависимые переменные. Аналогично представленному выше примеру с интерфейсом Coefficient Form PDE, это можно сделать, добавив ODE-интерфейс из раздела Mathematics. Предположим, например, что на каждом временном шаге требуется вычислять интеграл от величины общего теплового потока на промежутке от старта до текущего момента, который показывает накопленную энергию. Переменная для общего теплового потока рассчитывается в COMSOL автоматически и называется ht.tfluxMag. Интеграл может быть вычислен как дополнительная зависимая переменная с помощью узла Distributed ODE (Распределенное обыкновенное дифференциальное уравнение) интерфейса Domain ODEs and DAEs. Правой частью (источниковым членом) для доменного ОДЕ должна выступать подынтегральная функция, что и показано на иллюстрации ниже.


    Использование дополнительного ODE-интерфейса для вычисления интеграла по времени.

    В чем польза подобной техники? Полученный интеграл можно повторно использовать в других физических интерфейсах, поля в которых могут зависеть от накопленной в системе энергии. Более того, полученный резултат будет мгноменно доступен для всех видов постобработки, что удобнее и быстрее, чем использование встроенных операторов. Рекомендуем ознакомится с моделью Carbon Deposition in Hetereogeneous Catalysis (Образование сажевых отложений при гетерогенном катализе), в которой ОДЕ в области используется для вычисления пористости катализатора при наличии химических реакций в виде нестационарной полевой переменной.

    Вычисление интеграла от аналитических функций и выражений

    До сих пор мы демонстрировали, каким образом вычислять интеграл от искомых переменных в процессе расчета или при постобработке. Но не касались случая взятия интегралов от аналитических функций или выражений. Для этой операции в среде COMSOL доступен встроенный оператор integrate(expression, integration variable, lower bound, upper bound).

    Выражение может представлять собой любую одномерную функцию, например sin(x). При этом допускается включение дополнительных переменных, например sin(x*y). Второй параметр определяет, по какой переменной вычисляется интеграл. Например, integrate(sin(x*y),y,0,1) выдает функцию переменной x, потому что интегрирование выполняется только по переменной y. Обратите внимание, что данный оператор также может использоваться для работы с аналитическими функциями, заданными в узле Definitions (Определения) текущего компонента.


    Добавление аналитической функции.
    Вычисление интеграла от аналитической функции.

    Материалы для дальнейшего изучения

    Что такое интеграл? + Пример

    В математике мы говорим о двух типах интегралов. Определенные интегралы и неопределенные интегралы .

    Как правило, интеграл присваивает числа функциям таким образом, чтобы они могли описывать смещение, площадь, объем и даже вероятность.

    Определенные интегралы

    Этот тип интеграла относится к числовым значениям. Он используется в чистой математике, прикладной математике, статистике, естествознании и многом другом.b f (x) “d” x #, где

    # ромб f “называется подынтегральным выражением” #
    # ромб a и b “нижняя и верхняя границы” #
    # алмаз x “фиктивная переменная” #

    Вам может быть интересно, что означает # “d” x #. Формально это ничего не означает, а скорее сообщает вам, по какой переменной вы дифференцируете, или, в нашем случае, сообщает вам переменную интеграции.

    Когда мы говорим о площади, определяемой функцией # f # с осью x, мы имеем в виду чистую площадь .n # от # 0 # до # тау #; ну, это называется правилом власти. Существует множество различных формул для интегралов, о которых я не буду рассказывать в этом ответе. Это всего лишь очень общее представление о том, что такое интегралы.

    Неопределенные интегралы

    Они представлены в виде интегралов с оценками. Пусть # I # – неопределенный интеграл функции # f #.

    # I = int f (x) “d” x #

    Вы можете думать о неопределенных интегралах как об обобщениях определенных.

    Неопределенные интегралы, вместо того чтобы определяться площадями, объемами или чем-то еще, коррелируют с производными. Неопределенный интеграл функции # f # также называется первообразной и часто обозначается как #F (x) #.

    Фундаментальная теорема исчисления устраняет разрыв между функцией, ее производной и неопределенным интегралом. 2 #.b f (x) “d” x = F (b) -F (a) #

    Надеюсь, этот ответ не был слишком пугающим.

    Интегралы | Безграничное исчисление

    Нефтепродукты

    Первообразная – это дифференцируемая функция [латекс] F [/ латекс], производная которой равна [латекс] f [/ латекс] (то есть [латекс] F ‘= f [/ латекс]).

    Цели обучения

    Вычислить первообразную (также известную как неопределенный интеграл) для заданной функции

    Основные выводы

    Ключевые моменты
    • Процесс определения первообразных называется антидифференцированием, а его противоположная операция называется дифференцированием, то есть процессом нахождения производной.
    • Первообразные связаны с определенными интегралами через фундаментальную теорему исчисления: определенный интеграл функции на интервале равен разнице между значениями первообразной, вычисленной на концах интервала.
    • Графики первообразных данной функции представляют собой вертикальные перемещения друг друга, причем положение каждого графа зависит от значения константы [латекс] C [/ латекс]. {b} f (x) dx = F (b) – F (a)} [/ latex]

      Из-за этого правила каждую из бесконечного числа первообразных данной функции [latex] f [/ latex] иногда называют «общим интегралом» или «неопределенным интегралом» [latex] f [/ latex] и записывают с использованием интегрального символа без границ:

      [латекс] \ displaystyle {\ int f (x) dx} [/ латекс]

      Если [латекс] F [/ латекс] является первообразным [латекса] f [/ латекс], и функция [латекс] f [/ латекс] определена на некотором интервале, то все остальные первообразные [латекс] G [/ latex] of [latex] f [/ latex] отличается от [latex] F [/ latex] константой: существует число [latex] C [/ latex] такое, что [latex] G (x) = F (x ) + C [/ latex] для всех [latex] x [/ latex].2} [/ latex] в его естественном домене:

      [латекс] (- \ infty; 0) \ bigcup (0; \ infty) [/ latex].

      Площадь и расстояния

      Определенные интегралы используются во многих практических ситуациях, когда требуется вычисление расстояния, площади и объема. {b} f (x) dx [/ latex] неформально определяется как площадь области на плоскости [latex] xy [/ latex], ограниченная график [latex] f [/ latex], ось [latex] x [/ latex] и вертикальные линии [latex] x = a [/ latex] и [latex] x = b [/ latex], таким образом, что область над осью [latex] x [/ latex] прибавляется к сумме, а область под осью [latex] x [/ latex] вычитается из общей суммы.{b} f (x) dx = F (b) – F (a) [/ латекс].

    • Когда практическое приближение не дает достаточно точных результатов для вычислений расстояния, площади и объема, необходимо выполнить интегрирование.
    Ключевые термины
    • первообразная : неопределенный интеграл
    • интеграция : операция поиска области на плоскости [latex] xy [/ latex], связанной заданной функцией
    • определенный интеграл : интеграл функции между верхней и нижней границей

    Интеграция – важное понятие в математике и, вместе с обратным дифференцированием, одна из двух основных операций в исчислении.{b} f (x) dx = F (b) – F (a) [/ латекс]. Определенные интегралы появляются во многих практических ситуациях, требующих вычислений расстояния, площади и объема.

    Площадь

    Для начала рассмотрим кривую [латекс] y = f (x) [/ latex] между [latex] 0 [/ latex] и [latex] x = 1 [/ latex] с [latex] f (x) = \ sqrt {x}. [/ latex]

    Мы спрашиваем: «Какова площадь под функцией [latex] f [/ latex] в интервале от [latex] 0 [/ latex] до [latex] 1 [/ latex]? »И назовем эту (пока неизвестную) область интегралом [латекса] f [/ латекса].{1} \ sqrt {x} dx} [/ латекс]

    В первом приближении посмотрите на единичный квадрат, заданный сторонами [латекс] x = 0 [/ латекс] до [латекс] x = 1 [/ латекс], [латекс] y = f (0) = 0 [/ латекс] и [латекс] y = f (1) = 1 [/ latex]. Его площадь ровно [латекс] 1 [/ латекс]. Как бы то ни было, истинное значение интеграла должно быть несколько меньше. Уменьшение ширины прямоугольников аппроксимации должно дать лучший результат, поэтому мы пересечем интервал за пять шагов, используя точки аппроксимации [latex] 0 [/ latex], [latex] \ frac {1} {5} [/ latex ], [latex] \ frac {2} {5} [/ latex] и так далее, вплоть до [latex] 1 [/ latex].Установите коробку для каждого шага, используя правую конечную высоту каждой части кривой, получив таким образом [латекс] \ sqrt {\ frac {1} {5}} [/ latex], [latex] \ sqrt {\ frac {2} { 5}} [/ latex] и так далее, вплоть до [latex] \ sqrt {1} = 1 [/ latex]. Суммируя площади этих прямоугольников, мы получаем лучшее приближение для искомого интеграла, а именно:

    [латекс] \ displaystyle {\ sqrt {\ frac {1} {5}} \ left (\ frac {1} {5} – 0 \ right) + \ sqrt {\ frac {2} {5}} \ left (\ frac {2} {5} – \ frac {1} {5} \ right) + \ cdots + \ sqrt {\ frac {5} {5}} \ left (\ frac {5} {5} – \ гидроразрыв {4} {5} \ right) \ приблизительно 0.7497} [/ латекс]

    Обратите внимание, что мы берем конечную сумму многих значений функции [latex] f [/ latex], умноженную на разности двух последующих точек аппроксимации. Легко видеть, что приближение все еще слишком велико. Использование большего количества шагов дает более точное приближение, но никогда не будет точным: заменив подынтервалы [latex] 5 [/ latex] на двенадцать, как показано, мы получим приблизительное значение для площади [latex] 0,6203 [/ latex], что очень маленький. Ключевой идеей является переход от добавления конечного числа разностей точек аппроксимации, умноженных на их соответствующие значения функций, к использованию бесконечного числа мелких или бесконечно малых шагов.{1/2} dx = F (1) – F (0) = \ frac {2} {3}} [/ latex]

    Расстояние (определение длины дуги интегрированием)

    Если вам известна скорость [latex] v (t) [/ latex] объекта как функция времени, вы можете просто интегрировать [latex] v (t) [/ latex] во времени, чтобы вычислить расстояние, которое прошел объект. . Поскольку это эквивалентно оценке площади под изгибом [латекс] v (t) [/ латекс], мы не будем обсуждать это подробнее.

    Однако вы также можете использовать интегралы для вычисления длины – например, длину дуги, описываемую функцией [latex] y = f (x) [/ latex].8} \, dt} [/ латекс]

    Определенный интеграл : Определенный интеграл функции может быть представлен в виде области со знаком области, ограниченной ее графиком.

    Определенный интеграл

    Определенный интеграл – это площадь области на плоскости [latex] xy [/ latex], ограниченная графиком [latex] f [/ latex], осью [latex] x [/ latex] и вертикальные линии [латекс] x = a [/ латекс] и [латекс] x = b [/ латекс].

    Цели обучения

    Вычислить определенный интеграл функции по заданному интервалу

    Основные выводы

    Ключевые моменты
    • Интегрирование – важное понятие в математике и – вместе с обратным ему дифференцированием – одна из двух основных операций в исчислении.{b} f (x) dx = F (b) – F (a) [/ латекс].
    • Определенные интегралы появляются во многих практических ситуациях, и их фактическое вычисление важно в точном машиностроении (любой дисциплины), который требует точных и строгих значений.
    Ключевые термины
    • определенный интеграл : интеграл функции между верхней и нижней границей
    • интеграция : операция поиска области на плоскости [latex] xy [/ latex], связанной функцией
    • первообразная : неопределенный интеграл

    Интегрирование – важное понятие в математике и, вместе с обратным ему дифференцированием, одна из двух основных операций в исчислении.{b} f (x) dx [/ latex] неофициально определяется как область области на плоскости [latex] xy [/ latex], ограниченная графом [latex] f [/ latex], [ латекс] x [/ latex] -ось, а вертикальные линии [latex] x = a [/ latex] и [latex] x = b [/ latex], так что область над [latex] x [/ latex] -axis прибавляет к итоговому значению, а область под осью [latex] x [/ latex] вычитается из итогового значения. Такие интегралы называются определенными интегралами.

    Определенный интеграл : Определенный интеграл функции может быть представлен в виде области со знаком области, ограниченной ее графиком.

    Принципы интеграции были независимо сформулированы Исааком Ньютоном и Готфридом Лейбницем в конце 17 века. Через фундаментальную теорему исчисления, которую они независимо разработали, интегрирование связано с дифференцированием: если [latex] f [/ latex] – это непрерывная функция с действительными значениями, определенная на отрезке [latex] [a, b] [/ latex] ], то, как только первообразное [латекс] F [/ латекс] из [латекса] f [/ латекса] известно, определенный интеграл [латекс] f [/ латекс] по этому интервалу будет равен

    [латекс] \ displaystyle {\ int_ {a} ^ {b} f (x) dx = F (b) – F (a)} [/ latex]

    Определенные интегралы появляются во многих практических ситуациях.Если бассейн прямоугольной формы с плоским дном, то по его длине, ширине и глубине мы можем легко определить объем воды, который он может содержать (чтобы заполнить его), площадь его поверхности (чтобы покрыть его) и длина его края (чтобы закрепить его). Но если она овальная с закругленным дном, все эти величины требуют интегралов. Для таких тривиальных примеров может быть достаточно практических приближений, но точная инженерия (любой дисциплины) требует точных и строгих значений для этих элементов.

    Например, рассмотрим кривую [латекс] y = f (x) [/ latex] между 0 и x = 1 с [латексом] f (x) = \ sqrt {x}.{1} \ sqrt {x} dx [/ латекс]

    В первом приближении посмотрите на единичный квадрат, заданный сторонами [латекс] x = 0 [/ латекс] до [латекс] x = 1 [/ латекс], [латекс] y = f (0) = 0 [/ латекс] и [латекс] y = f (1) = 1 [/ latex]. Его площадь ровно [латекс] 1 [/ латекс]. Как бы то ни было, истинное значение интеграла должно быть несколько меньше. Уменьшение ширины прямоугольников аппроксимации должно дать лучший результат, поэтому мы пересечем интервал за пять шагов, используя точки аппроксимации [latex] 0 [/ latex], [latex] \ frac {1} {5} [/ latex ], [latex] \ frac {2} {5} [/ latex] и так далее, вплоть до [latex] 1 [/ latex].Установите коробку для каждого шага, используя правую конечную высоту каждой части кривой, получив таким образом [латекс] \ sqrt {\ frac {1} {5}} [/ latex], [latex] \ sqrt {\ frac {2} { 5}} [/ latex] и так далее, вплоть до [latex] \ sqrt {1} = 1 [/ latex]. Суммируя площади этих прямоугольников, мы получаем лучшее приближение для искомого интеграла, а именно:

    [латекс] \ displaystyle {\ sqrt {\ frac {1} {5}} \ left (\ frac {1} {5} – 0 \ right) + \ sqrt {\ frac {2} {5}} \ left (\ frac {2} {5} – \ frac {1} {5} \ right) + \ cdots + \ sqrt {\ frac {5} {5}} \ left (\ frac {5} {5} – \ гидроразрыв {4} {5} \ right) \ приблизительно 0.7497} [/ латекс]

    Обратите внимание, что мы берем конечную сумму многих значений функции [latex] f [/ latex], умноженную на разности двух последующих точек аппроксимации. Легко видеть, что приближение все еще слишком велико. Использование большего количества шагов дает более точное приближение, но никогда не будет точным: заменив подынтервалы [latex] 5 [/ latex] на двенадцать, как показано, мы получим приблизительное значение для площади [latex] 0,6203 [/ latex], что очень маленький. Ключевой идеей является переход от добавления конечного числа разностей точек аппроксимации, умноженных на их соответствующие значения функций, к использованию бесконечного числа мелких или бесконечно малых шагов.{1/2} dx = F (1) – F (0) = \ frac {2} {3}} [/ latex]

    Основная теорема исчисления

    Основная теорема исчисления – это теорема, которая связывает понятие производной функции с понятием интеграла.

    Цели обучения

    Определите первую и вторую фундаментальные теоремы исчисления

    Основные выводы

    Ключевые моменты
    • Первая часть теоремы показывает, что неопределенное интегрирование можно обратить дифференцированием.
    • Вторая часть позволяет вычислить определенный интеграл функции, используя любую из ее бесконечного множества первообразных.
    • Вторая часть теоремы имеет неоценимые практические применения, поскольку она заметно упрощает вычисление определенных интегралов.
    Ключевые термины
    • первообразная : неопределенный интеграл
    • производная : мера того, как функция изменяется при изменении ее входных данных
    • определенный интеграл : интеграл функции между верхней и нижней границей

    Основная теорема исчисления – это теорема, которая связывает понятие производной функции с понятием интеграла.

    Основная теорема исчисления : Из этого рисунка видно, что основная теорема исчисления работает. По определению, производная [latex] A (x) [/ latex] равна [latex] \ frac {A (x + h) −A (x)} {h} [/ latex] как [latex] h [/ latex] стремится к нулю. Заменив числитель [латекс] A (x + h) -A (x) [/ latex] на [latex] hf (x) [/ latex] и разделив на [latex] h [/ latex], [latex] ] f (x) [/ latex] получается. Ограничение [latex] h [/ latex] стремится к нулю завершает доказательство основной теоремы исчисления.

    Теорема состоит из двух частей. Грубо говоря, первая часть посвящена производной первообразной, а вторая часть касается взаимосвязи между первообразными и определенными интегралами.

    Первая часть теоремы, иногда называемая первой фундаментальной теоремой исчисления, показывает, что неопределенное интегрирование можно обратить дифференцированием. Эта часть теоремы важна еще и потому, что она гарантирует существование первообразных для непрерывных функций.{b} f (t) dt} [/ латекс]

    Теперь [латекс] F [/ латекс] непрерывен на [латекс] [a, b] [/ латекс], дифференцируемый на открытом интервале [латекс] (a, b) [/ латекс] и [латекс] F ‘(x) = f (x) [/ latex] для всех [latex] x [/ latex] в [latex] (a, b) [/ latex].

    Вторая часть, иногда называемая второй фундаментальной теоремой исчисления, позволяет вычислить определенный интеграл функции, используя любую из ее бесконечного числа первообразных. Эта часть теоремы имеет неоценимые практические приложения, поскольку она заметно упрощает вычисление определенных интегралов.{b} f (x) dx = F (b) – F (a)} [/ латекс]

    Первое опубликованное утверждение и доказательство ограниченной версии фундаментальной теоремы было сделано Джеймсом Грегори (1638–1675). Исаак Барроу (1630–1677) доказал более обобщенную версию теоремы, а ученик Барроу Исаак Ньютон (1643–1727) завершил развитие окружающей математической теории. Готфрид Лейбниц (1646–1716) систематизировал полученные знания в исчисление бесконечно малых величин и ввел обозначения, используемые сегодня. {b} f (x) dx = f (b) – f (a) [/ latex]

    Ключевые термины
    • первообразная : неопределенный интеграл
    • определенный интеграл : интеграл функции между верхней и нижней границей
    • интегральный : также иногда называют первообразным; предел сумм, вычисляемых в процессе, в котором область определения функции делится на небольшие подмножества, а возможное номинальное значение функции на каждом подмножестве умножается на меру этого подмножества, после чего все эти продукты суммируются

    Неопределенные интегралы и первообразные

    Как вы помните из атомов на первообразных, [латекс] F [/ латекс] считается первообразным от [латекса] f [/ латекса], если [латекс] F ‘(x) = f (x) [/ latex ].Однако [латекс] F [/ латекс] не единственное первообразное. Мы можем добавить любую константу [latex] C [/ latex] к [latex] F [/ latex] без изменения производной. Имея это в виду, мы определяем неопределенный интеграл следующим образом: [latex] \ int f (x) dx = F (x) + C [/ latex], где [latex] F [/ latex] удовлетворяет [latex] F ‘ (x) = f (x) [/ latex] и [latex] C [/ latex] – любая константа.

    [latex] f (x) [/ latex], интегрируемая функция, называется подынтегральным выражением. Обратите внимание, что неопределенный интеграл дает семейство функций.2} {2} \ right) -x + c [/ latex], показывая три из бесконечного множества решений, которые могут быть получены путем изменения произвольной константы [latex] C [/ latex].

    Неопределенные интегралы обладают следующими основными свойствами.

    Постоянное правило для неопределенных интегралов

    [латекс] \ int cf (x) dx = c \ int f (x) dx [/ латекс]

    Правило суммы для неопределенных интегралов

    [латекс] \ int (f (x) + g (x)) dx = \ int f (x) dx + \ int g (x) dx [/ latex]

    Правило разности неопределенных интегралов

    [латекс] \ int (f (x) – g (x)) dx = \ int f (x) dx – \ int g (x) dx [/ latex]

    Определенные интегралы и теорема чистого изменения

    Интегрирование по заданной области дает так называемый «определенный интеграл» (в том смысле, что область определена).{b} f (x) dx = f (b) – f (a)} [/ латекс]

    В основном теорема утверждает, что интеграл или [латекс] F ‘[/ латекс] от [латекса] a [/ латекса] до [латекса] b [/ латекса] представляет собой область между [латексом] a [/ латексом] и [latex] b [/ latex], или разница в площади от положения [latex] f (a) [/ latex] до положения [latex] f (b) [/ latex]. Это можно применять для поиска таких значений, как объем, концентрация, плотность, численность населения, стоимость и скорость.

    Правило замены

    Интегрирование подстановкой – важный инструмент математиков, используемый для нахождения интегралов и первообразных.

    Цели обучения

    Используйте [latex] u [/ latex] -замещение (правило замещения), чтобы найти первообразную более сложных функций

    Основные выводы

    Ключевые моменты
    • Замена [латекс] x = g (t) [/ latex] дает [latex] \ frac {dx} {dt} = g ‘(t) [/ latex] и, следовательно, формально [latex] dx = g ‘(t) dt [/ latex], который является необходимой заменой для [latex] dx [/ latex].
    • [latex] u [/ latex] -замещение (также называемое [latex] w [/ latex] -замещением) используется для упрощения данного интеграла.
    • Замещение можно использовать для определения первообразных.
    Ключевые термины
    • интеграция : операция поиска области в плоскости [latex] x [/ latex] – [latex] y [/ latex], связанной функцией
    • первообразная : неопределенный интеграл

    Интегрирование путем подстановки, также известное как [латекс] u [/ латекс] -замещение, представляет собой метод нахождения интегралов. Использование основной теоремы исчисления часто требует поиска первообразной.По этой и другим причинам интегрирование путем подстановки является важным инструментом для математиков. Это аналог цепного правила дифференциации.

    Определенный интеграл : Определенный интеграл функции может быть представлен в виде области со знаком области, ограниченной ее графиком.

    Пусть [latex] I \ substeq \ mathbb {R} [/ latex] будет интервалом, а [latex] g: [a, b] \ rightarrow 1 [/ latex] будет непрерывно дифференцируемой функцией. Предположим, что [latex] f: I \ rightarrow \ mathbb {R} [/ latex] – непрерывная функция.{b} f (g (t)) g ‘(t) dt} [/ латекс]

    Используя обозначения Лейбница, замена [latex] x = g (t) [/ latex] дает:

    [латекс] \ displaystyle {\ frac {dx} {dt} = g ‘(t)} [/ латекс]

    и, следовательно, формально:

    [латекс] dx = g ‘(t) dt [/ латекс]

    , который является необходимой заменой для [latex] dx [/ latex].

    Формула используется для преобразования одного интеграла в другой, который легче вычислить. Таким образом, формулу можно использовать слева направо или справа налево, чтобы упростить данный интеграл.2 + 1 = 5 [/ latex], преобразование обратно в термины [латекс] x [/ latex] было ненужным.

    Подстановка может использоваться для определения первообразных, если выбирается связь между [латексом] x [/ латексом] и [латексом] u [/ латексом], определяется соответствующая связь между [латексом] dx [/ латексом] и [латексом] du. [/ latex] путем дифференцирования и выполняет замены. Можно надеяться, что первообразная замещенной функции может быть определена; исходная замена между [латексом] u [/ латексом] и [латексом] x [/ латексом] отменяется.2 + 1 [/ латекс].

    Дополнительные трансцендентные функции

    Трансцендентная функция – это функция, не являющаяся алгебраической.

    Цели обучения

    Определить трансцендентную функцию как функцию, которая не может быть выражена как конечная последовательность алгебраической операции

    Основные выводы

    Ключевые моменты
    • Трансцендентные функции не могут быть выражены как решение полиномиального уравнения, коэффициенты которого сами являются полиномами с рациональными коэффициентами.
    • Примеры трансцендентных функций включают экспоненциальную функцию, логарифм и тригонометрические функции.
    • Трансцендентные функции могут быть легко обнаруживаемым источником размерных ошибок.
    Ключевые термины
    • полином : выражение, состоящее из суммы конечного числа членов, каждый член является произведением постоянного коэффициента и одной или нескольких переменных, возведенных в неотрицательную целую степень
    • тригонометрическая функция : любая функция угла, выраженная как отношение двух сторон прямоугольного треугольника, имеющего этот угол, или различные другие функции, которые вычитают 1 из этого значения или вычитают это значение из 1 (например, синус)
    • экспоненциальная функция : любая функция, в которой независимая переменная представлена ​​в форме экспоненты; они являются обратными функциями логарифмов

    Трансцендентная функция – это функция, не являющаяся алгебраической.Такая функция не может быть выражена как решение полиномиального уравнения, коэффициенты которого сами являются полиномами с рациональными коэффициентами. Примеры трансцендентных функций включают экспоненциальную функцию, логарифм и тригонометрические функции.

    Тригонометрические функции : Верхняя панель: Тригонометрическая функция sinθ для выбранных углов [латекс] \ тета [/ латекс], [латекс] \ пи – \ тета [/ латекс], [латекс] \ пи + \ тета [/ латекс] , и [латекс] 2 \ pi – \ theta [/ latex] в четырех квадрантах.Нижняя панель: График функции синуса в зависимости от угла.

    Формально аналитическая функция [latex] ƒ (z) [/ latex] вещественных или комплексных переменных [latex] z_1, \ cdots, z_n [/ latex] трансцендентна, если [latex] z_1, \ cdots, z_n [/ latex], [latex] ƒ (z) [/ latex] алгебраически независимы, т. е. если [latex] ƒ [/ latex] трансцендентен над полем [latex] C (z_1, \ cdots, z_n) [/ latex] .

    Трансцендентная функция – это функция, которая «превосходит» алгебру в том смысле, что она не может быть выражена в терминах конечной последовательности алгебраических операций сложения, умножения, степени и извлечения корня.x [/ latex] – трансцендентная функция. Аналогично, если мы установим [latex] c [/ latex] равным [latex] e [/ latex] в 5 , то мы обнаружим, что [latex] \ ln (x) [/ latex], натуральный логарифм, это трансцендентная функция.

    В анализе размерностей трансцендентные функции примечательны тем, что они имеют смысл только тогда, когда их аргумент безразмерен (возможно, после алгебраической редукции). Из-за этого трансцендентные функции могут быть легко обнаруживаемым источником размерных ошибок. Например, [latex] \ log (5 \ text {meter}) [/ latex] – бессмысленное выражение, в отличие от [latex] \ log \ left (\ frac {5 \ text {meter}} {3 \ text {meter }} \ right) [/ latex] или [latex] \ log (3) \ text {meter} [/ latex].Можно попытаться применить логарифмическую идентичность для получения [latex] \ log (10) + \ log (m) [/ latex], что подчеркивает проблему: применение неалгебраической операции к измерению дает бессмысленные результаты.

    Численное интегрирование

    Численное интегрирование – это метод приближения значения определенного интеграла. {b} f (x ) dx [/ latex] неофициально определяется как область области на плоскости [latex] xy [/ latex], ограниченная графом [latex] f [/ latex], [latex] x [/ latex] ] ось, а вертикальные линии [латекс] x = a [/ latex] и [latex] x = b [/ latex], так что область над осью [latex] x [/ latex] добавляется к общей , и что область под осью [latex] x [/ latex] вычитается из общей суммы.Эти интегралы называются «определенными интегралами».

    Определенный интеграл : Определенный интеграл функции может быть представлен в виде области со знаком области, ограниченной ее графиком.

    Численное интегрирование представляет собой широкое семейство алгоритмов для вычисления числового значения определенного интеграла.

    Есть несколько причин для проведения численного интегрирования.

    Подынтегральное выражение [латекс] f (x) [/ латекс] может быть известно только в определенных точках, например, полученных путем выборки.2) [/ latex], первообразная которого (функция ошибок, умноженная на константу) не может быть записана в элементарной форме. Может быть возможно найти первообразную символически, но может быть проще вычислить численное приближение, чем вычислить первообразную. Это может иметь место, если первообразная дана как бесконечная серия или произведение, или если для ее оценки требуется специальная функция, которая недоступна.

    Методы численного интегрирования обычно можно описать как объединение оценок подынтегрального выражения для получения приближения к интегралу.

    Подынтегральная функция вычисляется в конечном наборе точек, называемых точками интегрирования, и взвешенная сумма этих значений используется для аппроксимации интеграла. Точки интегрирования и веса зависят от конкретного используемого метода и требуемой точности аппроксимации. Важной частью анализа любого метода численного интегрирования является изучение поведения ошибки аппроксимации в зависимости от количества вычислений подынтегрального выражения. Метод, который дает небольшую ошибку при небольшом количестве оценок, обычно считается лучшим.Уменьшение количества вычислений подынтегрального выражения уменьшает количество задействованных арифметических операций и, следовательно, уменьшает общую ошибку округления. Кроме того, каждая оценка требует времени, и подынтегральное выражение может быть произвольно сложным. Численное интегрирование типа “грубой силы” может быть выполнено, если подынтегральное выражение имеет достаточно хорошее поведение (то есть кусочно-непрерывное и ограниченное изменение), путем вычисления подынтегрального выражения с очень маленькими приращениями.

    Самый простой метод численного решения интегралов – использовать правило средней точки или прямоугольника.af (x) dx \ приблизительно (b − a) f \ left (\ frac {a + b} {2} \ right)} [/ latex]

    Затем площадь можно приблизительно определить путем сложения площадей прямоугольников. Обратите внимание: чем меньше прямоугольники, тем точнее приближение.

    Правило прямоугольника : Иллюстрация правила численного интегрирования прямоугольника. {n -1} \ left (f \ left (a + k \ frac {ba} {n} \ right) \ right) + {f (b) \ over 2} \ right)} [/ latex]

    , где подынтервалы имеют вид [latex] [kh, (k + 1) h] [/ latex], где [latex] h = \ frac {(b − a)} {n} [/ latex] и [latex] ] k = 0, 1, 2, \ cdots, n − 1 [/ latex].

    Поскольку компьютеры могут выполнять множество арифметических операций за небольшой промежуток времени, они используют численное интегрирование для аппроксимации значений интегралов, а не решают их так, как это сделал бы человек.

    Исчисление I – Определение определенного интеграла

    Показать уведомление для мобильных устройств Показать все заметки Скрыть все заметки

    Похоже, вы используете устройство с “узкой” шириной экрана ( i.е. вы, вероятно, пользуетесь мобильным телефоном). Из-за особенностей математики на этом сайте лучше всего просматривать в ландшафтном режиме. Если ваше устройство не находится в альбомном режиме, многие уравнения будут отображаться сбоку от вашего устройства (вы сможете прокручивать их, чтобы увидеть их), а некоторые пункты меню будут обрезаны из-за узкой ширины экрана.

    Раздел 5-6: Определение определенного интеграла

    В этом разделе мы формально определим определенный интеграл и дадим многие свойства определенных интегралов.*} \ right) \ Delta x} \]

    Определенный интеграл определяется как предел и сумма, которые мы рассмотрели в предыдущем разделе, чтобы найти чистую площадь между функцией и осью \ (x \). Также обратите внимание, что обозначение для определенного интеграла очень похоже на обозначение для неопределенного интеграла. Причина этого со временем станет очевидной.

    Есть также небольшая терминология, от которой мы должны избавиться. Число «\ (a \)», которое находится внизу знака интеграла, называется нижним пределом интеграла, а число «\ (b \)» в верхней части знака интеграла называется верхним пределом . предел интеграла.Кроме того, несмотря на то, что \ (a \) и \ (b \) были заданы как интервал, нижний предел не обязательно должен быть меньше верхнего. В совокупности мы часто будем называть \ (a \) и \ (b \) интервал интегрирования .

    Давайте быстро рассмотрим пример. В этом примере будут использоваться многие свойства и факты из краткого обзора нотации суммирования в главе «Дополнительно».

    Пример 1 Используя определение определенного интеграла, вычислите следующее.* \). Чтобы облегчить себе жизнь, мы будем использовать правильные конечные точки каждого интервала.

    Из предыдущего раздела мы знаем, что для общего \ (n \) ширина каждого подынтервала составляет

    \ [\ Delta x = \ frac {{2 – 0}} {n} = \ frac {2} {n} \]

    Тогда подынтервалы равны

    . 3}}} + \ frac {2}) {n}} \ right)} \ end {align *} \]

    Теперь нам нужно взять предел этого.Значит, нам нужно будет «оценить» это суммирование. Другими словами, нам придется использовать формулы, приведенные в обзоре нотации суммирования, чтобы исключить фактическое суммирование и получить формулу для этого общего \ (n \).

    Для этого нам нужно признать, что \ (n \) является константой в том, что касается обозначения суммирования. По мере того, как мы перебираем целые числа от 1 до \ (n \) в суммировании, изменяется только \ (i \), и поэтому все, что не является \ (i \), будет постоянным и может быть исключено из суммирования.2}}} \\ & = \ frac {{14}} {3} \ end {align *} \]

    Мы видели несколько методов решения этой проблемы с ограничением, поэтому оставим вам проверять результаты.

    Вау, это было много работы для довольно простой функции. Есть гораздо более простой способ их оценки, и в конце концов мы до него доберемся. Основная цель этого раздела – разъяснить основные свойства и факты об определенном интеграле. Мы обсудим, как мы вычисляем их на практике, начиная со следующего раздела.{{\, b}} {{f \ left (t \ right) \, dt}} \). Смысл этого свойства в том, чтобы заметить, что, пока функция и пределы совпадают, переменная интегрирования, которую мы используем в определенном интеграле, не повлияет на ответ.

    Доказательство свойств 1–4 см. В разделе «Доказательство различных интегральных свойств» в главе «Дополнительные возможности». Свойство 5 доказать непросто, поэтому оно здесь не показано. Свойство 6 не является собственностью в полном смысле этого слова. Это сделано только для того, чтобы признать, что до тех пор, пока функция и ограничения одинаковы, не имеет значения, какую букву мы используем для переменной.{{\, 12}} {{f \ left (x \ right) \, dx}} \). Показать решение

    Этот пример в основном является примером свойства 5, хотя в решении также есть несколько вариантов использования свойства 1.

    Нам нужно выяснить, как правильно разбить интеграл, используя свойство 5, чтобы мы могли использовать заданные фрагменты информации. Во-первых, отметим, что существует интеграл, в одном из пределов которого стоит «-5». Это не нижний предел, но мы можем использовать свойство 1, чтобы в конечном итоге это исправить.{{\, b}} {{\ left | {f \ left (x \ right) \,} \ right | dx}} \)

    См. Доказательство этих свойств в разделе «Доказательство различных интегральных свойств» в главе «Дополнительные возможности».

    Интерпретации определенного интеграла

    Здесь мы можем дать несколько быстрых интерпретаций определенного интеграла.

    Во-первых, как мы упоминали в предыдущем разделе, одна из возможных интерпретаций определенного интеграла состоит в том, чтобы дать чистую площадь между графиком \ (f \ left (x \ right) \) и осью \ (x \) на интервал \ (\ left [{a, b} \ right] \).{{\, b}} {{f ‘\ left (x \ right) \, dx}} = f \ left (b \ right) – f \ left (a \ right) \]

    – чистое изменение \ (f \ left (x \ right) \) на интервале \ (\ left [{a, b} \ right] \). {{\, {t_2}}} {{v \ left (t \ right) \, dt}} = s \ left ({{t_2}} \ right ) – s \ left ({{t_1}} \ right) \]

    Обратите внимание, что в этом случае if \ (v \ left (t \ right) \) одновременно положительно и отрицательно ( i.{{\, {t_2}}} {{\ left | {v \ left (t \ right)} \ right | \, dt}} \]

    Здесь важно отметить, что теорема чистого изменения имеет смысл только в том случае, если мы интегрируем производную функции.

    Фундаментальная теорема исчисления, часть I

    Как указано в названии выше, это только первая часть фундаментальной теоремы исчисления. Мы дадим вторую часть в следующем разделе, так как это ключ к простому вычислению определенных интегралов, и это тема следующего раздела.2} + 1}} \, dt}} \) Показать решение

    Над этим нужно немного поработать, прежде чем мы сможем использовать фундаментальную теорему исчисления. Первое, на что следует обратить внимание, это то, что основная теорема исчисления требует, чтобы нижний предел был константой, а верхний предел – переменной. {{\, v \ left (x \ right)}} {{f \ left (t \ right) \, dt }} = – v ‘\ left (x \ right) f \ left ({v \ left (x \ right)} \ right) \]

    Наконец, мы также можем получить версию для обоих пределов, являющихся функциями \ (x \).2}} \ right) \ end {align *} \]

    Определенные интегралы

    Определенный интеграл функции тесно связан с первообразной и неопределенным интегралом функции. Основное отличие состоит в том, что неопределенный интеграл, если он существует, является вещественным числовым значением, в то время как последние два представляют бесконечное количество функций, которые отличаются только константой. Взаимосвязь между этими понятиями будет обсуждаться в разделе, посвященном фундаментальной теореме исчисления, и вы увидите, что определенный интеграл будет иметь приложения ко многим задачам исчисления.

    Развитие определения определенного интеграла начинается с функции f ( x ), которая непрерывна на отрезке [ a, b ]. Данный интервал разбивается на подинтервалы « n », которые, хотя и не обязательно, могут быть взяты равными по длине (Δ x ). Произвольное значение домена, x i , выбирается в каждом подынтервале, и определяется его последующее значение функции, f ( x i ).Определяется произведение каждого значения функции на соответствующую длину подынтервала, и эти произведения « n » складываются для определения их суммы. Эта сумма называется суммой Римана и может быть положительной, отрицательной или нулевой, в зависимости от поведения функции на закрытом интервале. Например, если f ( x )> 0 на [ a, b ], тогда сумма Римана будет положительным действительным числом. Если f ( x ) <0 на [ a, b ], тогда сумма Римана будет отрицательным действительным числом.Сумма Римана функции f ( x ) на [ a, b ] выражается как

    Таким образом, сумму Римана можно представить как «сумму n произведений».

    Пример 1: Вычислите сумму Римана для f ( x ) = x 2 на [1,3], используя четыре подинтервала одинаковой длины, где x i – правое конечная точка в подынтервале и (см. рисунок).

    Рисунок 1 Сумма Римана с четырьмя частями.

    Поскольку подынтервалы должны иметь одинаковую длину, получается, что

    Сумма Римана для четырех подинтервалов равна

    .

    Если количество подинтервалов многократно увеличивать, то в результате длина каждого подынтервала будет становиться все меньше и меньше. Это можно переформулировать следующим образом: если количество подынтервалов неограниченно увеличивается ( n → + ∞), то длина каждого подынтервала приближается к нулю (Δ x → + ∞).Этот предел суммы Римана, если он существует, используется для определения определенного интеграла функции на [ a, b ]. Если f ( x ) определено на закрытом интервале [ a, b ], то определенный интеграл из f ( x ) от a до b определяется как

    , если это ограничение выходит за пределы.

    Функция f ( x ) называется подынтегральным выражением, а переменная x является переменной интегрирования.Числа a, и b, называются пределами интегрирования, а a, – нижним пределом интегрирования, а b, – верхним пределом интегрирования.

    Обратите внимание, что символ ∫, используемый с неопределенным интегралом, – это тот же символ, который ранее использовался для неопределенного интеграла функции. Причина этого станет более очевидной в следующем обсуждении фундаментальной теоремы исчисления. Также имейте в виду, что определенный интеграл является уникальным действительным числом и не представляет бесконечное количество функций, которые являются результатом неопределенного интеграла функции.

    Вопрос о существовании предела суммы Римана важно рассмотреть, поскольку он определяет, существует ли определенный интеграл для функции на отрезке. Как и в случае с дифференцированием, между непрерывностью и интегрированием существует значительная взаимосвязь, которую можно резюмировать следующим образом: если функция f ( x ) непрерывна на отрезке [ a, b ], то определенный интеграл от f ( x ) на [ a, b ] существует, а f считается интегрируемым на [ a, b ].Другими словами, непрерывность гарантирует, что определенный интеграл существует, но обратное не обязательно верно.

    К сожалению, тот факт, что определенный интеграл функции существует на отрезке, не означает, что значение определенного интеграла легко найти.

    Некоторые свойства полезны при решении задач, требующих применения определенного интеграла. Некоторые из наиболее распространенных свойств:

    1.

    2.

    3., где c – постоянная

    4.

    5. Правило суммы:

    6. Правило разницы:

    7. Если

    8. Если

    9. Если

    10. Если a, b, и c – любые три точки на закрытом интервале, то

    11. Теорема о среднем значении для определенных интегралов: если f ( x ) непрерывно на закрытом интервале [ a, b ], то по крайней мере одно число c существует в открытом интервале ( a , b ) такая, что

    Значение f ( c ) называется средним или средним значением функции f ( x ) на интервале [ a, b ] и

    .

    Пример 2: Оценить

    Пример 3: Учитывая, что

    Пример 4: Учитывая, что

    Пример 5 Вычислить

    Пример 6: Учитывая, что оценка

    Пример 7: Учитывая, что оцените.

    Пример 8: Учитывая это, оцените.

    Пример 9: При условии, что найти все значения c , которые удовлетворяют теореме о среднем значении для данной функции на отрезке.

    По теореме о среднем значении

    Поскольку находится в интервале (3,6), вывод теоремы о среднем значении выполняется для этого значения c .

    Основная теорема исчисления

    Фундаментальная теорема исчисления устанавливает связь между неопределенными и определенными интегралами и вводит методику вычисления определенных интегралов без использования сумм Римана, что очень важно, поскольку вычисление предела суммы Римана может быть чрезвычайно трудоемким и трудным.Утверждение теоремы таково: если f ( x ) непрерывно на интервале [ a, b ], а F ( x ) является любой первообразной f ( x ) по [ a, b ], затем

    Другими словами, значение определенного интеграла функции на [ a, b ] – это разность любой первообразной функции, вычисленной на верхнем пределе интегрирования, за вычетом той же первообразной, вычисленной на нижнем пределе интегрирования.Поскольку константы интегрирования одинаковы для обеих частей этой разности, они игнорируются при вычислении определенного интеграла, поскольку они вычитаются и дают ноль. Помня об этом, выберите постоянную интегрирования равной нулю для всех определенных интегральных вычислений после примера 10.

    Пример 10: Оценить

    Поскольку общая первообразная x 2 равна (1/3) x 3 + C , вы обнаружите, что

    Пример 11: Оценить

    Поскольку первообразная sin x – cos x , вы обнаружите, что

    Пример 12: Оценить

    (Потому что, (первообразная от, и вы обнаружите, что

    Пример 13: Оценить

    Поскольку первообразное x 2 – 4 x + 1 равно (1/3) x 3 – 2 x 2 + x , вы обнаружите, что

    Определенная интегральная оценка

    Многочисленные методы, которые можно использовать для вычисления неопределенных интегралов, также могут использоваться для вычисления определенных интегралов.Методы подстановки и замены переменных, интегрирования по частям, тригонометрических интегралов и тригонометрической подстановки проиллюстрированы в следующих примерах.

    Пример 14: Оценить

    Используя метод замены с

    пределы интегрирования могут быть преобразованы из значений x в соответствующие им значения и . Когда x = 1, u = 3 и когда x = 2, u = 6, вы обнаружите, что

    Обратите внимание, что когда метод подстановки используется для вычисления определенных интегралов, нет необходимости возвращаться к исходной переменной, если пределы интегрирования преобразуются в новые значения переменных.

    Пример 15: Оценить

    Используя метод подстановки с u = sin x + 1, du = cos x dx , вы обнаружите, что u = 1, когда x = π, и u = 0, когда x = 3π / 2; следовательно,

    Обратите внимание, что вам никогда не приходилось возвращаться к тригонометрическим функциям в исходном интеграле для вычисления определенного интеграла.

    Пример 16: Оценить

    Использование интеграции по частям с

    вы обнаружите, что

    Пример 17: Оценить

    Использование интеграции по частям с

    Пример 18: Оценить

    Пример 19: Оценить.

    Пример 20: Оценить.

    Поскольку подынтегральное выражение содержит форму a 2 + x 2 ,

    Рисунок 2 Диаграмма для примера 20.

    Пример 21: Оценить

    Поскольку радикал имеет вид

    Рисунок 3 Диаграмма для примера 21.

    Расчет

    – Что такое интеграция $ dx $?

    Интеграл дает вам площадь между горизонтальной осью и кривая. В большинстве случаев это ось x.

      г
    
                             | |
                           - | - ---- | ---- f (x)
                         / | \ / |
                        / | -------- |
              | / | |
         ----- | ------- | |
              | | |
              | | |
    ---------- | -------------- + -------------------- | --- -- Икс
            а б
      

    И закрытая площадь:

    Площадь = $ \ int ^ b_a f (x) dx $

    Но предположим, что вы не хотите использовать интеграл для измерения область между осью x и кривой.Вместо этого ты просто вычислить среднее значение графика между a и b и нарисуйте полосатую ровную линию y = avg (x) (среднее значение x в этом диапазоне).

    Теперь у вас есть такой график:

      г
    
                             | |
                           - | - - - | - - f (x)
              | / | \ / |
         ----- | ----------------------------------- | ---- средн. (x)
              | / | |
         - - - | - - - - | |
              | | |
              | | |
    ---------- | -------------- + -------------------- | --- -- Икс
            а б
      

    А заключенная область представляет собой прямоугольник:

    Площадь = avg (x) w, где w – ширина i секции

    Высота равна avg (x), а ширина – w = b-a или на английском языке, “ширина среза оси x от a до b.«

    Но скажем, вам нужна более точная площадь. Вы можете сломать разбейте график на более мелкие участки и сделайте из них прямоугольники. Допустим, вы делаете 4 равных секции:

      г
    
                               | |
                          | ---- | --- | | ------- | ---- f (x)
                          | | | | |
                          | | | -------- | |
                | | | | | |
           ----- | --------- | | | | |
                | | | | | |
                | | | | | |
      ---------- | --------- | ---- + --- | -------- | ------- | --- -- Икс
                а б
      

    А площадь составляет:

    Площадь = раздел 1 + раздел 2 + раздел 3 + раздел 4

    = средн (x, 1) w + avg (x, 2) w + avg (x, 3) w + avg (x, 4) w

    где w – ширина каждой секции.4_ {n = 1} avg (x, n) w $

    Но все равно недостаточно точно. Давайте использовать бесконечное количество разделов. Теперь наш район становится суммой бесконечного числа разделов. Поскольку это бесконечная сумма, мы будем использовать знак интеграла вместо знака суммы:

    Площадь = $ \ int avg (x) w $

    где avg (x) для бесконечно тонкого сечения будет равным f (x) в этом разделе, а w будет “шириной бесконечно тонкий участок оси x ».

    Итак, вместо avg (x) мы можем написать f (x), потому что они то же самое, если взять среднее по бесконечно малой ширине.

    И мы можем переименовать переменную w во все, что захотим. Ширина секции – это разница между правая сторона и левая сторона. Разница между две точки часто называют дельтой этих значений. Таким образом, разница двух значений x (например, a и b) будет называться дельта-х. Но это слишком долго, чтобы использовать уравнение, поэтому, когда у нас есть бесконечно малая дельта, он сокращается до dx.

    Если мы заменим avg (x) и w на эти эквивалентные значения:

    Площадь = $ \ int f (x) dx $

    Итак, уравнение говорит следующее:

    Площадь равна сумме бесконечного количества прямоугольников. которые имеют высоту f (x) и ширину dx (где dx – бесконечно небольшое расстояние).

    Значит, вам нужен dx, потому что иначе вы не суммируете прямоугольники вверх, и ваш ответ не будет общей площадью.

    dx буквально означает «бесконечно малая ширина x».

    Это даже означает это в производных. Производная от функция – это наклон графика в этой точке. Наклон обычно измеряется как разность y двух баллов, разделенных на разность этих баллов x:

    Наклон = (y2 – y1) / (x2 – x1)

    Но чем ближе становятся эти точки, тем меньше они различия получаются.Начнем называть их дельтами, потому что разница между двумя точками часто бывает называется дельтой этих значений.

    Наклон = дельта-y / дельта-x

    Дельты становятся все меньше и меньше по мере того, как эти два x, y точки становятся все ближе и ближе. Когда они бесконечно малое расстояние друг от друга, то дельта-у а delta-x сокращается до dy и dx:

    Наклон = dy / dx

    Наклон все еще остается Наклон = (y2 – y1) / (x2 – x1) но эти точки бесконечно близки друг к другу, поэтому мы используем dy и dx, чтобы сказать себе, что они бесконечно близкие или «дифференциальные расстояния».«

    Что мне нужно знать об интегралах?

    Вводная статья к этому «Интегралы: математический смысл» была довольно длинной, утомительной и математической. В этом классе мы не будем просить вас делать такие обширные математические выводы. Итак, что мы хотим, чтобы вы знали?

    Основы

    Вот несколько основных моментов, которые вам нужно знать об интеграле.

    1. Интеграл – это сумма произведений функции ($ g $) на небольшое изменение ($ dx $).
    2. Суммируя произведение $ g $ на $ dx $, мы создаем функцию (назовем ее $ f $), производная которой равна $ g $.
    3. Поскольку интеграл представляет собой сумму произведений, размерность интеграла будет произведением размерности $ g $, умноженной на размерность $ dx $.
    4. Обычно интеграл дает только форму функции, производная которой равна $ g $. Поскольку вы можете добавить константу к $ f $, и новая функция будет иметь ту же производную (поскольку производная константы равна 0), чтобы получить функцию, которая представлена ​​интегралом, вы должны знать значение функции вы создаете в одной точке ($ f (x_0) $).х $.
    5. Интеграл от $ \ cos {x} $ равен $ \ sin {x} $, а интеграл от $ \ sin {x} $ равен $ – \ cos {x} $
    6. Есть несколько полезных способов рассмотрения интегралов:

      • Интеграл как продукт
      • Интеграл как шаговое правило
      • Интеграл на графике

      Интеграл как продукт

      Ключевые уравнения для понимания интеграла – умножение на определение производной (помните, $ g = \ frac {df} {dx} $), а затем открытие изменения в $ f $ и решение для окончательного значения.Они записываются в виде трех уравнений, которые легко следуют одно из следующего.

      $$ g = \ frac {\ Delta f} {\ Delta x} \ rightarrow \ Delta f = g \ Delta x $$

      $$ f_f – f_i \ приблизительно g (x_i) \ Delta x $$

      $$ f_f \ приблизительно f_i + g (x_i) \ Delta x $$

      , где мы используем индекс «i» для обозначения «начального значения» и «f» для обозначения «конечного значения». Это не так уж и плохо. На самом деле это просто алгебра (с множеством символов, как это принято в науке!).

      Уравнение выше говорит, что небольшое изменение в $ f $ = небольшое изменение, $ g \ Delta x $.Интеграл просто складывается с этими небольшими изменениями, чтобы получить большое изменение.

      Интеграл как шаговое правило

      После того, как у нас появился продукт, в приведенных выше уравнениях мы просто открыли $ \ Delta f $, чтобы показать его как явное изменение. Затем мы переместили начальное значение вправо. Таким образом, правая сторона зависит только от начальной точки и дает нам окончательное значение в ближайшей точке. Если, например, наша «$ g $» означает ускорение, поскольку ускорение является производной скорости, тогда скорость должна быть интегралом ускорения.Если мы знаем ускорение как функцию времени (для любого времени) и знаем начальное значение скорости, мы можем генерировать скорости в более позднее время. Тройка приведенных выше уравнений теперь будет выглядеть так:

      $$ a = \ frac {\ Delta v} {\ Delta t} \ rightarrow \ Delta v = a \ Delta t $$

      $$ v_f – v_i \ приблизительно a (t_i) \ Delta t $$

      $$ v_f \ приблизительно v_i + a (t_i) \ Delta t $$

      Мы можем получить скорость в более позднее время из скорости и ускорения в более раннее время – предсказывая будущее путем интегрирования! Этот метод оказался очень ценным способом решения проблемы движения объектов с использованием нашего фундаментального принципа движения, 2-го закона Ньютона.(См. Ньютон 2 как правило шага.)

      Интеграл в виде графика и отношение к производной

      Мы также можем смотреть на интеграл как на что-то на графике, как на производную. Например, скорость ($ v $) является производной положения ($ x $) по времени, $ v = dx / dt $. Это означает, что положение является интегралом скорости по времени, $ dx = v dt $. Обратите внимание, что интеграл дает нам только изменение функции, которую он создает. Чтобы получить все, вам нужно знать начальное значение.

      Эти идеи представлены на следующих графиках с соответствующими уравнениями.

      Первый набор представляет, как получить график производной (v) от графика функции (x), взяв наклон («маленькие треугольники» или «подъем за пробегом»)

      Эти уравнения показывают, что происходит:

      $$ v (t) = \ frac {dx} {dt} $$

      $$ v (t) = \ frac {x (t + \ frac {\ Delta t} {2}) – x (t- \ frac {\ Delta t} {2})} {\ Delta t} $$

      Начнем с графика x-t.Каждый раз на графике x-t (5 показаны пунктирными линиями) мы рисуем маленькие треугольники, показывающие, как изменяется x в течение небольшого временного интервала вокруг каждого выбранного момента. Отношение этих двух изменений и есть скорость в данный момент.

      Чтобы вернуться назад – от скорости к положению – мы должны интегрироваться. Этот процесс представлен этими графиками.

      Эти уравнения показывают, что происходит:

      $$ dx = v (t) dt $$

      $$ \ Delta x = \ sum {dx} = \ int {v (t) dt} $$

      Сопоставляя уравнения с графиком, мы начинаем с графика v в определенное время и выбираем небольшой интервал времени, $ dt $.Произведение средней скорости в этом интервале на $ dt $ – это площадь показанного маленького прямоугольника. Эта область имеет размеры [$ v dt $] = (L / T) (T) = L, так что это изменение положения. Если у нас есть начальное значение $ x $ в начальный момент, мы можем использовать область на графике vt, чтобы выяснить изменение $ x $ и, следовательно, какое новое значение $ x $ будет в конце временной интервал.

      Используя пример положения и скорости, пара уравнений

      $$ v = \ frac {\ Delta x} {\ Delta t} \ quad \ quad \ Delta x = v \ Delta t $$

      показывает, как думать о переходе от графика функции к графику производных и обратно.Производная – это отношение (так что … “рост за пробег” или “маленькие треугольники”), а интеграл – это произведение (поэтому площадь равна изменению).

      Джо Редиш 9/4/11

      5.2 Определенный интеграл – Исчисление Том 1

      Цели обучения

      • 5.2.1 Сформулируйте определение определенного интеграла.
      • 5.2.2 Объясните термины подынтегральное выражение, пределы интегрирования и переменная интегрирования.
      • 5.2.3 Объясните, когда функция интегрируема.
      • 5.2.4 Опишите взаимосвязь между определенным целым и чистой площадью.
      • 5.2.5 Используйте геометрию и свойства определенных интегралов для их вычисления.
      • 5.2.6 Вычислить среднее значение функции.

      В предыдущем разделе мы определили площадь под кривой в терминах сумм Римана:

      A = limn → ∞∑i = 1nf (xi *) Δx.A = limn → ∞∑i = 1nf (xi *) Δx.

      Однако это определение имело ограничения. Мы потребовали, чтобы f (x) f (x) была непрерывной и неотрицательной.К сожалению, реальные проблемы не всегда соответствуют этим ограничениям. В этом разделе мы рассмотрим, как применить концепцию площади под кривой к более широкому набору функций с помощью определенного интеграла.

      Определение и обозначения

      Определенный интеграл обобщает понятие площади под кривой. Мы снимаем требования о непрерывности и неотрицательности функции f (x) f (x) и определяем определенный интеграл следующим образом.

      Определение

      Если f (x) f (x) – функция, определенная на интервале [a, b], [a, b], определенный интеграл f от a до b равен

      ∫abf (x) dx = limn → ∞∑i = 1nf (xi *) Δx, ∫abf (x) dx = limn → ∞∑i = 1nf (xi *) Δx,

      (5.8)

      при наличии ограничения. Если этот предел существует, функция f (x) f (x) называется интегрируемой на [a, b], [a, b] или интегрируемой функцией.

      Знак интеграла в предыдущем определении должен показаться знакомым. Мы видели аналогичные обозначения в главе «Применение производных», где мы использовали символ неопределенного интеграла (без a и b сверху и снизу) для представления первообразной. Хотя обозначения для неопределенных интегралов могут выглядеть аналогично обозначениям для определенного интеграла, они не совпадают.Определенный интеграл – это число. Неопределенный интеграл – это семейство функций. Позже в этой главе мы исследуем, как связаны эти концепции. Однако всегда следует уделять пристальное внимание обозначениям, чтобы мы знали, работаем ли мы с определенным интегралом или с неопределенным интегралом.

      Интегральная система обозначений восходит к концу семнадцатого века и является одним из вкладов Готфрида Вильгельма Лейбница, которого часто считают соавтором исчисления вместе с Исааком Ньютоном.Символ интегрирования ∫ представляет собой удлиненную букву S, обозначающую сигму или суммирование. На определенном интеграле выше и ниже символа суммирования находятся границы интервала, [a, b]. [A, b]. Числа a и b являются значениями x и называются пределами интегрирования; в частности, a – это нижний предел, а b – верхний предел. Чтобы уточнить, мы используем слово limit двумя разными способами в контексте определенного интеграла.Сначала поговорим о пределе суммы при n → ∞.n → ∞. Во-вторых, границы области называются пределами интеграции .

      Мы называем функцию f (x) f (x) подынтегральным выражением, а dx указывает, что f (x) f (x) является функцией относительно x , называемой переменной интегрирования. Обратите внимание, что, как и индекс в сумме, переменная интегрирования является фиктивной переменной и не влияет на вычисление интеграла. В качестве переменной интегрирования мы можем использовать любую понравившуюся переменную:

      Abf (x) dx = ∫abf (t) dt = ∫abf (u) du∫abf (x) dx = ∫abf (t) dt = ∫abf (u) du

      Ранее мы обсуждали тот факт, что если f (x) f (x) непрерывна на [a, b], [a, b], то предел limn → ∞∑i = 1nf (xi *) ∆xlimn → ∞∑ i = 1nf (xi *) Δx существует и единственно.Это приводит к следующей теореме, которую мы сформулируем без доказательства.

      Теорема 5.1

      Непрерывные функции интегрируемы

      Если f (x) f (x) непрерывна на [a, b], [a, b], то f интегрируема на [a, b]. [A , б].

      Функции, которые не являются непрерывными на [a, b] [a, b], могут быть интегрируемыми, в зависимости от характера разрывов. Например, функции с конечным числом скачков на отрезке интегрируемы.

      Здесь также стоит отметить, что мы сохранили использование регулярного разбиения в суммах Римана.Это ограничение не является строго необходимым. Любое разбиение можно использовать для образования суммы Римана. Однако, если для определения определенного интеграла используется нерегулярное разбиение, этого недостаточно, поскольку количество подынтервалов стремится к бесконечности. Вместо этого мы должны принять предел, так как ширина самого большого подинтервала стремится к нулю. Это вводит немного более сложные обозначения в наши пределы и усложняет вычисления без особого дополнительного понимания, поэтому мы придерживаемся регулярных разбиений для сумм Римана.

      Пример 5.7

      Вычисление интеграла по определению

      Используйте определение определенного интеграла для вычисления ∫02x2dx.∫02x2dx. Используйте аппроксимацию правой конечной точки, чтобы получить сумму Римана.

      Решение

      Сначала мы хотим установить сумму Римана. Исходя из пределов интегрирования, имеем a = 0a = 0 и b = 2.b = 2. Для i = 0,1,2,…, n, i = 0,1,2,…, n, пусть P = {xi} P = {xi} – регулярное разбиение [0,2]. [0, 2]. Тогда

      Δx = b − an = 2n.Δx = b − an = 2n.

      Поскольку мы используем аппроксимацию правой конечной точки для генерации сумм Римана, для каждого i нам нужно вычислить значение функции на правом конце интервала [xi − 1, xi]. [Xi − 1, xi] . Правая конечная точка интервала – xi, xi, и, поскольку P является обычным разделом,

      xi = x0 + iΔx = 0 + i [2n] = 2in.xi = x0 + iΔx = 0 + i [2n] = 2in.

      Таким образом, значение функции на правом конце интервала равно

      . f (xi) = xi2 = (2in) 2 = 4i2n2.f (xi) = xi2 = (2in) 2 = 4i2n2.

      Тогда сумма Римана принимает вид

      ∑i = 1nf (xi) Δx = ∑i = 1n (4i2n2) 2n = ∑i = 1n8i2n3 = 8n3∑i = 1ni2.∑i = 1nf (xi) Δx = ∑i = 1n (4i2n2) 2n = ∑i = 1n8i2n3 = 8n3∑i = 1ni2.

      Используя формулу суммирования для ∑i = 1ni2, ∑i = 1ni2, получаем

      ∑i = 1nf (xi) Δx = 8n3∑i = 1ni2 = 8n3 [n (n + 1) (2n + 1) 6] = 8n3 [2n3 + 3n2 + n6] = 16n3 + 24n2 + 8n6n3 = 83 + 4n + 86n2.∑i = 1nf (xi) Δx = 8n3∑i = 1ni2 = 8n3 [n (n + 1) (2n + 1) 6] = 8n3 [2n3 + 3n2 + n6] = 16n3 + 24n2 + 8n6n3 = 83 + 4n + 86n2.

      Теперь, чтобы вычислить определенный интеграл, нам нужно перейти к пределу при n → ∞.n → ∞. Получаем

      ∫02x2dx = limn → ∞∑i = 1nf (xi) Δx = limn → ∞ (83 + 4n + 86n2) = limn → ∞ (83) + limn → ∞ (4n) + limn → ∞ (86n2) = 83 + 0. + 0 = 83. 02x2dx = limn → ∞∑i = 1nf (xi) ∆x = limn → ∞ (83 + 4n + 86n2) = limn → ∞ (83) + limn → ∞ (4n) + limn → ∞ (86n2 ) = 83 + 0 + 0 = 83.

      КПП 5,7

      Используйте определение определенного интеграла, чтобы вычислить ∫03 (2x − 1) dx.∫03 (2x − 1) dx. Используйте аппроксимацию правой конечной точки, чтобы получить сумму Римана.

      Вычисление определенных интегралов

      Вычисление определенных интегралов таким способом может быть довольно утомительным из-за сложности вычислений. Позже в этой главе мы разработаем методы вычисления определенных интегралов без взятия пределов сумм Римана. Однако пока мы можем полагаться на тот факт, что определенные интегралы представляют площадь под кривой, и мы можем вычислять определенные интегралы, используя геометрические формулы для вычисления этой площади.Мы делаем это, чтобы подтвердить, что определенные интегралы действительно представляют собой площади, поэтому мы можем обсудить, что делать в случае, если кривая функции опускается ниже оси x .

      Пример 5.8

      Использование геометрических формул для вычисления определенных интегралов

      Используйте формулу площади круга, чтобы вычислить ∫369− (x − 3) 2dx.369− (x − 3) 2dx.

      Решение

      Функция описывает полукруг с радиусом 3. Найти

      ∫369− (x − 3) 2dx, ∫369− (x − 3) 2dx,

      мы хотим найти площадь под кривой на интервале [3,6].[3,6]. Формула для вычисления площади круга: A = πr2.A = πr2. Площадь полукруга составляет половину площади круга, или A = (12) πr2.A = (12) πr2. Заштрихованная область на рис. 5.16 покрывает половину полукруга, или A = (14) πr2.A = (14) πr2. Таким образом,

      ∫369− (x − 3) 2 = 14π (3) 2 = 94π≈7,069. 369− (x − 3) 2 = 14π (3) 2 = 94π≈7,069. Рис. 5.16. Значение интеграла функции f (x) f (x) на интервале [3,6] [3,6] представляет собой площадь заштрихованной области.

      КПП 5.8

      Используйте формулу площади трапеции, чтобы вычислить ∫24 (2x + 3) dx.∫24 (2x + 3) dx.

      Площадь и определенный интеграл

      При определении определенного интеграла мы сняли требование неотрицательности f (x) f (x). Но как мы интерпретируем «площадь под кривой», когда f (x) f (x) отрицательно?

      Чистая подписанная площадь

      Вернемся к сумме Римана. Рассмотрим, например, функцию f (x) = 2−2x2f (x) = 2−2×2 (показанную на рисунке 5.17) на интервале [0,2]. [0,2]. Используйте n = 8n = 8 и выберите {xi *} {xi *} в качестве левой конечной точки каждого интервала.Постройте прямоугольник на каждом подынтервале высотой f (xi *) f (xi *) и шириной Δ x . Когда f (xi *) f (xi *) положительно, произведение f (xi *) Δxf (xi *) Δx, как и раньше, представляет площадь прямоугольника. Однако, когда f (xi *) f (xi *) отрицательно, произведение f (xi *) Δxf (xi *) Δx представляет отрицательных площади прямоугольника. Сумма Римана тогда становится

      . ∑i = 18f (xi *) Δx = (Площадь прямоугольников над осью x) – (Площадь прямоугольников ниже оси x) ∑i = 18f (xi *) Δx = (Площадь прямоугольников над осью x) – ( Площадь прямоугольников ниже оси x)

      Рисунок 5.17 Для частично отрицательной функции сумма Римана – это площадь прямоугольников над осью x минус площадь прямоугольников под осью x .

      Принимая предел при n → ∞, n → ∞, сумма Римана приближается к площади между кривой над осью x и осью x за вычетом площади между кривой под осью x и ось x , как показано на рис. 5.18. Затем

      ∫02f (x) dx = limn → ∞∑i = 1nf (ci) Δx = A1 − A2.∫02f (x) dx = limn → ∞∑i = 1nf (ci) Δx = A1 − A2.

      Величина A1 − A2A1 − A2 называется чистой подписанной областью.

      Рисунок 5.18 В пределе определенный интеграл равен площади A 1 минус площадь A 2 или чистая подписанная площадь.

      Обратите внимание, что чистая подписанная площадь может быть положительной, отрицательной или нулевой. Если область над осью x больше, чистая подписанная область положительна. Если область под осью x больше, чистая подписанная область отрицательна.Если области выше и ниже оси x равны, чистая подписанная область равна нулю.

      Пример 5.9

      Поиск чистой подписанной области

      Найдите чистую площадь со знаком между кривой функции f (x) = 2xf (x) = 2x и осью x на интервале [−3,3]. [- 3,3].

      Решение

      Функция создает прямую линию, которая образует два треугольника: один от x = −3x = −3 до x = 0x = 0, а другой от x = 0x = 0 до x = 3x = 3 (рисунок 5.19). Используя геометрическую формулу для площади треугольника, A = 12bh, A = 12bh, площадь треугольника A 1 над осью равна

      .

      , где 3 – основание, а 2 (3) = 62 (3) = 6 – высота. Площадь треугольника A 2 ниже оси равна

      . A2 = 12 (3) (6) = 9, A2 = 12 (3) (6) = 9,

      , где 3 – это основание, а 6 – высота. Таким образом, чистая площадь составляет

      . ∫ − 332xdx = A1 − A2 = 9−9 = 0. − 332xdx = A1 − A2 = 9−9 = 0.

      Рис. 5.19. Площадь над кривой и под осью x равна площади под кривой и над осью x .

      Анализ

      Если A 1 – это область выше оси x , а A 2 – область ниже оси x , то чистая площадь равна A1 − A2.A1 − A2. Поскольку площади двух треугольников равны, чистая площадь равна нулю.

      КПП 5.9

      Найдите чистую знаковую площадь f (x) = x − 2f (x) = x − 2 в интервале [0,6], [0,6], как показано на следующем рисунке.

      Общая площадь

      Одно из применений определенного интеграла – определение смещения при заданной функции скорости.Если v (t) v (t) представляет скорость объекта как функцию времени, тогда площадь под кривой сообщает нам, насколько далеко объект от своего исходного положения. Это очень важное применение определенного интеграла, и мы рассмотрим его более подробно позже в этой главе. А пока мы просто рассмотрим некоторые основы, чтобы понять, как это работает, изучая постоянные скорости.

      Когда скорость постоянна, площадь под кривой равна скорости, умноженной на время. Эта идея уже хорошо знакома.Если автомобиль удаляется от исходного положения по прямой со скоростью 70 миль в час в течение 2 часов, то он находится на расстоянии 140 миль от исходного положения (рис. 5.20). В интегральных обозначениях имеем

      ∫0270dt = 140.∫0270dt = 140. Рис. 5.20. Площадь под кривой v (t) = 75v (t) = 75 показывает, как далеко автомобиль находится от начальной точки в данный момент времени.

      В контексте смещения чистая подписанная площадь позволяет нам учитывать направление. Если автомобиль движется прямо на север со скоростью 60 миль в час в течение 2 часов, он находится в 120 милях к северу от своей начальной позиции.Если после этого автомобиль развернется и поедет на юг со скоростью 40 миль в час в течение 3 часов, он вернется в исходное положение (рис. 5.21). Опять же, используя интегральные обозначения, имеем

      ∫0260dt + ∫25−40dt = 120−120 = 0. 0260dt + ∫25−40dt = 120−120 = 0.

      В этом случае смещение равно нулю.

      Рисунок 5.21 Площадь над осью и площадь под осью равны, поэтому чистая подписанная площадь равна нулю.

      Предположим, мы хотим знать, как далеко автомобиль проехал в целом, независимо от направления.В этом случае мы хотим знать область между кривой и осью x , независимо от того, находится эта область выше или ниже оси. Это называется общей площадью.

      С графической точки зрения проще всего рассчитать общую площадь, добавив области над осью и области под осью (вместо вычитания областей под осью, как мы это делали с чистой подписанной областью). Чтобы достичь этого математически, мы используем функцию абсолютного значения. Таким образом, общее расстояние, пройденное автомобилем, составит

      . 02 | 60 | dt + ∫25 | −40 | dt = ∫0260dt + ∫2540dt = 120 + 120 = 240.02 | 60 | dt + ∫25 | −40 | dt = ∫0260dt + ∫2540dt = 120 + 120 = 240.

      Формально объединяя эти идеи, мы даем следующие определения.

      Определение

      Пусть f (x) f (x) – интегрируемая функция, определенная на интервале [a, b]. [A, b]. Пусть A 1 представляет область между f (x) f (x) и осью x , которая лежит на выше оси , и пусть A 2 представляет область между f (x) f (x) и ось x , которая находится на ниже оси .Тогда чистая подписанная область между f (x) f (x) и осью x будет равна

      . Abf (x) dx = A1 − A2. Abf (x) dx = A1 − A2.

      Общая площадь между f (x) f (x) и осью x равна

      Ab | f (x) | dx = A1 + A2. Ab | f (x) | dx = A1 + A2.

      Пример 5.10

      Определение общей площади

      Найдите общую площадь между f (x) = x − 2f (x) = x − 2 и осью x на интервале [0,6]. [0,6].

      Решение

      Вычислите точку пересечения x как (2,0) (2,0) (установите y = 0, y = 0, найдите x ).Чтобы найти общую площадь, возьмите область ниже оси x на подынтервале [0,2] [0,2] и добавьте ее к области над осью x на подынтервале [2,6] [2,6] (Рисунок 5.22).

      Рис. 5.22. Общая площадь между линией и осью x на [0,6] [0,6] составляет A 2 плюс A 1 .

      У нас

      ∫06 | (x − 2) | dx = A2 + A1. 06 | (x − 2) | dx = A2 + A1.

      Тогда, используя формулу площади треугольника, получим

      A2 = 12bh = 12 · 2 · 2 = 2 A2 = 12bh = 12 · 2 · 2 = 2 A1 = 12bh = 12 · 4 · 4 = 8.A1 = 12bh = 12 · 4 · 4 = 8.

      Таким образом, общая площадь составляет

      A1 + A2 = 8 + 2 = 10. A1 + A2 = 8 + 2 = 10.

      КПП 5.10

      Найдите общую площадь между функцией f (x) = 2xf (x) = 2x и осью x на интервале [−3,3]. [- 3,3].

      Свойства определенного интеграла

      Свойства неопределенных интегралов применимы также к определенным интегралам. Определенные интегралы также имеют свойства, относящиеся к пределам интегрирования. Эти свойства вместе с правилами интегрирования, которые мы рассмотрим далее в этой главе, помогают нам манипулировать выражениями для вычисления определенных интегралов.

      Правило: свойства определенного интеграла


      1. ∫aaf (x) dx = 0∫aaf (x) dx = 0

        (5.9)


        Если пределы интегрирования одинаковы, интеграл представляет собой просто линию и не содержит области.

      2. ∫baf (x) dx = −∫abf (x) dx∫baf (x) dx = −∫abf (x) dx

        (5.10)


        Если пределы поменяны местами, поставьте знак минус перед интеграл.

      3. ab [f (x) + g (x)] dx = ∫abf (x) dx + ∫abg (x) dx∫ab [f (x) + g (x)] dx = ∫abf (x) dx + ∫abg (x) dx

        (5.11)


        Интеграл от суммы – это сумма интегралов.

      4. ab [f (x) −g (x)] dx = ∫abf (x) dx − ∫abg (x) dx∫ab [f (x) −g (x)] dx = ∫abf (x) dx − ∫abg (x) dx

        (5.12)


        Интеграл разности – это разность интегралов.

      5. ∫abcf (x) dx = c∫abf (x) ∫abcf (x) dx = c∫abf (x)

        (5.13)


        для константы c . Интеграл от произведения константы и функции равен константе, умноженной на интеграл функции.

      6. ∫abf (x) dx = ∫acf (x) dx + ∫cbf (x) dx∫abf (x) dx = ∫acf (x) dx + ∫cbf (x) dx

        (5.14)


        Хотя эта формула обычно применяется, когда c находится между a и b , формула верна для всех значений a , b и c , при условии, что f (x) f ( x) интегрируемо на наибольшем интервале.

      Пример 5.11

      Использование свойств определенного интеграла

      Используйте свойства определенного интеграла, чтобы выразить определенный интеграл от f (x) = – 3×3 + 2x + 2f (x) = – 3×3 + 2x + 2 в интервале [−2,1] [- 2,1] как сумму трех определенных интегралов.

      Решение

      В интегральных обозначениях имеем ∫ − 21 (−3×3 + 2x + 2) dx.∫ − 21 (−3×3 + 2x + 2) dx. Применяем свойства 3. и 5., чтобы получить

      . ∫ − 21 (−3×3 + 2x + 2) dx = ∫ − 21−3x3dx + ∫ − 212xdx + ∫ − 212dx = −3∫ − 21x3dx + 2∫ − 21xdx + ∫ − 212dx.∫ − 21 (−3×3 + 2x + 2) dx = ∫ − 21−3x3dx + ∫ − 212xdx + ∫ − 212dx = −3∫ − 21x3dx + 2∫ − 21xdx + ∫ − 212dx.

      КПП 5.11

      Используйте свойства определенного интеграла, чтобы выразить определенный интеграл от f (x) = 6×3−4×2 + 2x − 3f (x) = 6×3−4×2 + 2x − 3 на интервале [1,3] [1,3] как сумму четырех определенных интегралов.

      Пример 5.12

      Использование свойств определенного интеграла

      Если известно, что ∫08f (x) dx = 10∫08f (x) dx = 10 и ∫05f (x) dx = 5, ∫05f (x) dx = 5, найдите значение ∫58f (x) dx.∫58f (x) dx.

      Решение

      В собственности 6.,

      Abf (x) dx = ∫acf (x) dx + ∫cbf (x) dx.∫abf (x) dx = ∫acf (x) dx + ∫cbf (x) dx.

      Таким образом,

      ∫08f (x) dx = ∫05f (x) dx + ∫58f (x) dx10 = 5 + ∫58f (x) dx5 = ∫58f (x) dx.∫08f (x) dx = ∫05f (x) dx + ∫ 58f (x) dx10 = 5 + ∫58f (x) dx5 = ∫58f (x) dx.

      Контрольно-пропускной пункт 5.12

      Если известно, что ∫15f (x) dx = −3∫15f (x) dx = −3 и ∫25f (x) dx = 4, ∫25f (x) dx = 4, найдите значение ∫12f ( x) dx.∫12f (x) dx.

      Сравнительные свойства интегралов

      Изображение иногда может рассказать нам о функции больше, чем результаты вычислений. Сравнение функций по их графикам, а также по их алгебраическим выражениям часто может дать новое понимание процесса интеграции. Интуитивно можно сказать, что если функция f (x) f (x) находится над другой функцией g (x), g (x), то область между f (x) f (x) и осью x больше площади между g (x) g (x) и осью x .Это верно в зависимости от интервала, в течение которого производится сравнение. Свойства определенных интегралов действительны независимо от того, a b.a> b. Однако следующие свойства относятся только к случаю a≤b, a≤b и используются, когда мы хотим сравнить размеры интегралов.

      Теорема 5.2

      Теорема сравнения
      1. Если f (x) ≥0f (x) ≥0 для a≤x≤b, a≤x≤b, то
        Abf (x) dx≥0.∫abf (x) dx≥0.
      2. Если f (x) ≥g (x) f (x) ≥g (x) для a≤x≤b, a≤x≤b, то
        ∫abf (x) dx≥∫abg (x) dx.∫abf (x) dx≥∫abg (x) dx.
      3. Если m и M – константы такие, что m≤f (x) ≤Mm≤f (x) ≤M для a≤x≤b, a≤x≤b, то
        m (b − a) ≤∫abf (x) dx≤M (b − a). m (b − a) ≤∫abf (x) dx≤M (b − a).

      Пример 5.13

      Сравнение двух функций за заданный интервал

      Сравните f (x) = 1 + x2f (x) = 1 + x2 и g (x) = 1 + xg (x) = 1 + x в интервале [0,1]. [0,1].

      Решение

      Построение графиков этих функций необходимо, чтобы понять, как они сравниваются в интервале [0,1].[0,1]. Первоначально при построении графика на графическом калькуляторе f (x) f (x) оказывается везде выше g (x) g (x). Однако на интервале [0,1], [0,1] графики кажутся поверх друг друга. Нам нужно увеличить масштаб, чтобы увидеть, что на интервале [0,1], g (x) [0,1], g (x) находится выше f (x) .f (x). Две функции пересекаются в точках x = 0x = 0 и x = 1x = 1 (рисунок 5.23).

      Рисунок 5.23 (a) Функция f (x) f (x) появляется над функцией g (x) g (x) за исключением интервала [0,1] [0,1] (b) Просмотр того же графика с больший масштаб показывает это более четко.

      Из графика видно, что на интервале [0,1] g (x) ≥f (x). [0,1], g (x) ≥f (x). Сравнивая интегралы по заданному интервалу [0,1], [0,1], мы также видим, что ∫01g (x) dx≥∫01f (x) dx∫01g (x) dx≥∫01f (x) dx ( Рисунок 5.24). Тонкая заштрихованная область показывает, насколько велика разница между этими двумя интегралами в интервале [0,1]. [0,1].

      Рисунок 5.24 (a) График показывает, что на интервале [0,1] g (x) ≥f (x), [0,1], g (x) ≥f (x), где равенство выполняется только на конечные точки интервала.(b) Просмотр того же графика с большим увеличением показывает это более четко.

      Среднее значение функции

      Нам часто нужно найти среднее значение набора чисел, например, среднюю оценку за тест. Предположим, вы получили следующие результаты тестов в своем классе алгебры: 89, 90, 56, 78, 100 и 69. Ваша семестровая оценка – это ваше среднее значение результатов теста, и вы хотите знать, какую оценку ожидать. Мы можем найти среднее значение, сложив все оценки и разделив их на количество оценок. В этом случае есть шесть результатов теста.Таким образом,

      89 + 90 + 56 + 78 + 100 + 696 = 4826≈80,33,89 + 90 + 56 + 78 + 100 + 696 = 4826≈80,33.

      Таким образом, ваша средняя оценка за тест составляет примерно 80,33, что соответствует B- в большинстве школ.

      Однако предположим, что у нас есть функция v (t) v (t), которая дает нам скорость объекта в любой момент времени t , и мы хотим найти среднюю скорость объекта. Функция v (t) v (t) принимает бесконечное количество значений, поэтому мы не можем использовать только что описанный процесс. К счастью, мы можем использовать определенный интеграл, чтобы найти среднее значение такой функции, как эта.

      Пусть f (x) f (x) непрерывна на интервале [a, b] [a, b] и пусть [a, b] [a, b] разделен на n подинтервалов шириной ∆x = (b −a) /n.Δx= (b − a) / n. Выберите представителя xi * xi * в каждом подынтервале и вычислите f (xi *) f (xi *) для i = 1,2,…, n.i = 1,2,…, n. Другими словами, рассматривайте каждый f (xi *) f (xi *) как выборку функции на каждом подынтервале. Среднее значение функции может быть приблизительно равно

      . f (x1 *) + f (x2 *) + ⋯ + f (xn *) n, f (x1 *) + f (x2 *) + ⋯ + f (xn *) n,

      , что по сути является тем же выражением, которое используется для вычисления среднего дискретных значений.

      Но мы знаем Δx = b − an, Δx = b − an, поэтому n = b − aΔx, n = b − aΔx, и мы получаем

      f (x1 *) + f (x2 *) + ⋯ + f (xn *) n = f (x1 *) + f (x2 *) + ⋯ + f (xn *) (b − a) Δx.f (x1 *) + f (x2 *) + ⋯ + f (xn *) n = f (x1 *) + f (x2 *) + ⋯ + f (xn *) (b − a) Δx.

      Следуя алгебре, числитель представляет собой сумму, которая представлена ​​как ∑i = 1nf (xi *), ∑i = 1nf (xi *), и мы делим на дробь. Чтобы разделить на дробь, переверните знаменатель и умножьте. Таким образом, приблизительное значение среднего значения функции равно

      . ∑i = 1nf (xi *) (b − a) Δx = (Δxb − a) ∑i = 1nf (xi *) = (1b − a) ∑i = 1nf (xi *) Δx.∑i = 1nf (xi *) (b − a) Δx = (Δxb − a) ∑i = 1nf (xi *) = (1b − a) ∑i = 1nf (xi *) Δx.

      Это сумма Римана. Затем, чтобы получить точное среднее значение , возьмите предел, поскольку n стремится к бесконечности. Таким образом, среднее значение функции равно

      . 1b − alimn → ∞∑i = 1nf (xi) Δx = 1b − a∫abf (x) dx. 1b − alimn → ∞∑i = 1nf (xi) Δx = 1b − a∫abf (x) dx.

      Определение

      Пусть f (x) f (x) непрерывна на интервале [a, b]. [A, b]. Тогда среднее значение функции f (x) f (x) (или f ave ) на [a, b] [a, b] равно

      fave = 1b − a∫abf (x) dx.fave = 1b − a∫abf (x) dx.

      Пример 5.14

      Нахождение среднего значения линейной функции

      Найдите среднее значение f (x) = x + 1f (x) = x + 1 на интервале [0,5]. [0,5].

      Решение

      Сначала изобразите функцию на указанном интервале, как показано на Рисунке 5.25.

      Рисунок 5.25 На графике показана площадь под функцией f (x) = x + 1f (x) = x + 1 над [0,5]. [0,5].

      Область представляет собой трапецию, лежащую на боку, поэтому мы можем использовать формулу площади для трапеции A = 12h (a + b), A = 12h (a + b), где h представляет высоту, а a и b представляют две параллельные стороны.Затем

      ∫05x + 1dx = 12h (a + b) = 12 · 5 · (1 + 6) = 352. 05x + 1dx = 12h (a + b) = 12 · 5 · (1 + 6) = 352.

      Таким образом, среднее значение функции равно

      . 15−0∫05x + 1dx = 15 · 352 = 72,15−0∫05x + 1dx = 15 · 352 = 72.

      КПП 5.13

      Найдите среднее значение f (x) = 6−2xf (x) = 6−2x в интервале [0,3]. [0,3].

      Раздел 5.2. Упражнения

      В следующих упражнениях пределы выражаются в виде интегралов.

      60.

      limn → ∞∑i = 1n (xi *) Δxlimn → ∞∑i = 1n (xi *) Δx по [1,3] [1,3]

      61.

      limn → ∞∑i = 1n (5 (xi *) 2−3 (xi *) 3) ∆xlimn → ∞∑i = 1n (5 (xi *) 2−3 (xi *) 3) ∆x над [0, 2] [0,2]

      62.

      limn → ∞∑i = 1nsin2 (2πxi *) Δxlimn → ∞∑i = 1nsin2 (2πxi *) Δx на [0,1] [0,1]

      63.

      limn → ∞∑i = 1ncos2 (2πxi *) Δxlimn → ∞∑i = 1ncos2 (2πxi *) Δx на [0,1] [0,1]

      В следующих упражнениях, учитывая L n или R n , как указано, выразите свои пределы как n → ∞n → ∞ как определенные интегралы, определяя правильные интервалы.

      64.

      Ln = 1n∑i = 1ni − 1nLn = 1n∑i = 1ni − 1n

      65.

      Rn = 1n∑i = 1ninRn = 1n∑i = 1nin

      66.

      Ln = 2n∑i = 1n (1 + 2i − 1n) Ln = 2n∑i = 1n (1 + 2i − 1n)

      67.

      Rn = 3n∑i = 1n (3 + 3 дюйма) Rn = 3n∑i = 1n (3 + 3 дюйма)

      68.

      Ln = 2πn∑i = 1n2πi − 1ncos (2πi − 1n) Ln = 2πn∑i = 1n2πi − 1ncos (2πi − 1n)

      69.

      Rn = 1n∑i = 1n (1 + дюйм) журнал ((1 + дюйм) 2) Rn = 1n∑i = 1n (1 + дюйм) журнал ((1 + дюйм) 2)

      В следующих упражнениях вычислите интегралы функций, изображенных на графике, с помощью формул для площадей треугольников и кругов и вычитания площадей под осью x .

      70. 72. 74.

      В следующих упражнениях вычислите интеграл, используя формулы площади.

      76.

      ∫03 (3 − x) dx∫03 (3 − x) dx

      77.

      ∫23 (3 − x) dx∫23 (3 − x) dx

      78.

      ∫ − 33 (3− | x |) dx∫ − 33 (3− | x |) dx

      79.

      ∫06 (3− | x − 3 |) dx∫06 (3− | x − 3 |) dx

      80.

      ∫ − 224 − x2dx∫ − 224 − x2dx

      81.

      ∫154− (x − 3) 2dx∫154− (x − 3) 2dx

      82.

      ∫01236− (x − 6) 2dx∫01236− (x − 6) 2dx

      83.

      ∫ − 23 (3− | x |) dx∫ − 23 (3− | x |) dx

      В следующих упражнениях используйте средние значения в левой ( L ) и правой ( R ) конечных точках, чтобы вычислить интегралы кусочно-линейных функций с графиками, которые проходят через данный список точек через указанные интервалы.

      84.

      {(0,0), (2,1), (4,3), (5,0), (6,0), (8,3)} {(0,0), (2,1) , (4,3), (5,0), (6,0), (8,3)} больше [0,8] [0,8]

      85.

      {(0,2), (1,0), (3,5), (5,5), (6,2), (8,0)} {(0,2), (1,0) , (3,5), (5,5), (6,2), (8,0)} больше [0,8] [0,8]

      86.

      {(−4, −4), (- 2,0), (0, −2), (3,3), (4,3)} {(- 4, −4), (- 2,0 ), (0, −2), (3,3), (4,3)} над [−4,4] [- 4,4]

      87.

      {(−4,0), (- 2,2), (0,0), (1,2), (3,2), (4,0)} {(- 4,0), (- 2,2), (0,0), (1,2), (3,2), (4,0)} больше [−4,4] [- 4,4]

      Предположим, что ∫04f (x) dx = 5∫04f (x) dx = 5 и ∫02f (x) dx = −3, ∫02f (x) dx = −3 и ∫04g (x) dx = −1 ∫04g (x) dx = −1 и ∫02g (x) dx = 2.∫02g (x) dx = 2. В следующих упражнениях вычислите интегралы.

      88.

      ∫04 (е (x) + g (x)) dx∫04 (f (x) + g (x)) dx

      89.

      ∫24 (е (x) + g (x)) dx∫24 (f (x) + g (x)) dx

      90.

      ∫02 (f (x) −g (x)) dx∫02 (f (x) −g (x)) dx

      91.

      ∫24 (f (x) −g (x)) dx∫24 (f (x) −g (x)) dx

      92.

      ∫02 (3f (x) −4g (x)) dx∫02 (3f (x) −4g (x)) dx

      93.

      ∫24 (4f (x) −3g (x)) dx∫24 (4f (x) −3g (x)) dx

      В следующих упражнениях используйте тождество − AAf (x) dx = ∫ − A0f (x) dx + ∫0Af (x) dx∫ − AAf (x) dx = ∫ − A0f (x) dx + ∫0Af (x) dx для вычисления интегралов.

      94.

      ∫ − ππsint1 + t2dt∫ − ππsint1 + t2dt (Подсказка: sin (−t) = – sin (t)) (Подсказка: sin (−t) = – sin (t))

      95.

      ∫ − ππt1 + costdt∫ − ππt1 + costdt

      В следующих упражнениях найдите чистую зону со знаком между f (x) f (x) и осью x.

      96.

      ∫13 (2 − x) dx∫13 (2 − x) dx ( Подсказка: Посмотрите на график f .)

      97.

      ∫24 (x − 3) 3dx∫24 (x − 3) 3dx ( Подсказка: Посмотрите на график f .)

      В следующих упражнениях, учитывая, что ∫01xdx = 12, ∫01x2dx = 13, ∫01xdx = 12, ∫01x2dx = 13 и ∫01x3dx = 14, ∫01x3dx = 14, вычислите интегралы.

      98.

      ∫01 (1 + x + x2 + x3) dx∫01 (1 + x + x2 + x3) dx

      99.

      ∫01 (1 − x + x2 − x3) dx∫01 (1 − x + x2 − x3) dx

      100.

      ∫01 (1 − x) 2dx∫01 (1 − x) 2dx

      101.

      ∫01 (1-2x) 3dx∫01 (1-2x) 3dx

      102.

      ∫01 (6x − 43×2) dx∫01 (6x − 43×2) dx

      103.

      ∫01 (7−5×3) dx∫01 (7−5×3) dx

      В следующих упражнениях используйте теорему сравнения.

      104.

      Покажите, что ∫03 (x2−6x + 9) dx≥0.∫03 (x2−6x + 9) dx≥0.

      105.

      Покажите, что ∫ − 23 (x − 3) (x + 2) dx≤0.∫ − 23 (x − 3) (x + 2) dx≤0.

      106.

      Покажите, что ∫011 + x3dx≤∫011 + x2dx.∫011 + x3dx≤∫011 + x2dx.

      107.

      Покажите, что ∫121 + xdx≤∫121 + x2dx.∫121 + xdx≤∫121 + x2dx.

      108.

      Докажите, что ∫0π / 2sintdt≥π4.∫0π / 2sintdt≥π4. (Подсказка: sint≥2tπ (Подсказка: sint≥2tπ над [0, π2]) [0, π2])

      109.

      Докажите, что ∫ − π / 4π / 4costdt≥π2 / 4.∫ − π / 4π / 4costdt≥π2 / 4.

      В следующих упражнениях найдите среднее значение f ave из f между a и b и найдите точку c , где f (c) = fave.f (c) = любимый.

      110.

      f (x) = x2, a = −1, b = 1 f (x) = x2, a = −1, b = 1

      111.

      f (x) = x5, a = −1, b = 1 f (x) = x5, a = −1, b = 1

      112.

      f (x) = 4 − x2, a = 0, b = 2f (x) = 4 − x2, a = 0, b = 2

      113.

      f (x) = (3− | x |), a = −3, b = 3f (x) = (3− | x |), a = −3, b = 3

      114.

      f (x) = sinx, a = 0, b = 2πf (x) = sinx, a = 0, b = 2π

      115.

      f (x) = cosx, a = 0, b = 2πf (x) = cosx, a = 0, b = 2π

      В следующих упражнениях приблизьте среднее значение, используя суммы Римана L 100 и R 100 .Как ваш ответ соотносится с точным данным ответом?

      116.

      [T] y = ln (x) y = ln (x) на интервале [1,4]; [1,4]; точное решение ln (256) 3−1.ln (256) 3−1.

      117.

      [T] y = ex / 2y = ex / 2 в интервале [0,1]; [0,1]; точное решение 2 (e − 1) .2 (e − 1).

      118.

      [T] y = tanxy = tanx в интервале [0, π4]; [0, π4]; точное решение 2ln (2) π.2ln (2) π.

      119.

      [T] y = x + 14 − x2y = x + 14 − x2 в интервале [−1,1]; [- 1,1]; точное решение – π6.π6.

      В следующих упражнениях вычислите среднее значение, используя левую сумму Римана L N для N = 1,10,100.N = 1,10,100. Как точность соотносится с заданным точным значением?

      120.

      [T] y = x2−4y = x2−4 в интервале [0,2]; [0,2]; точное решение -83.-83.

      121.

      [T] y = xex2y = xex2 в интервале [0,2]; [0,2]; точное решение – 14 (e4−1) .14 (e4−1).

      122.

      [T] y = (12) xy = (12) x в интервале [0,4]; [0,4]; точное решение – 1564ln (2).1564ln (2).

      123.

      [T] y = xsin (x2) y = xsin (x2) в интервале [−π, 0]; [- π, 0]; точное решение – cos (π2) −12π.cos (π2) −12π.

      124.

      Предположим, что A = ∫02πsin2tdtA = ∫02πsin2tdt и B = ∫02πcos2tdt.B = ∫02πcos2tdt. Покажем, что A + B = 2πA + B = 2π и A = B.A = B.

      125.

      Предположим, что A = ∫ − π / 4π / 4sec2tdt = πA = ∫ − π / 4π / 4sec2tdt = π и B = ∫ − π / 4π / 4tan2tdt.B = ∫ − π / 4π / 4tan2tdt. Покажем, что A − B = π2.A − B = π2.

      126.

      Покажите, что среднее значение sin2tsin2t на [0,2π] [0,2π] равно 1/2 Без дальнейших вычислений определите, равно ли среднее значение sin2tsin2t на [0, π] [0, π] до 1/2.

      127.

      Покажите, что среднее значение cos2tcos2t на [0,2π] [0,2π] равно 1 / 2,1 / 2. Без дополнительных вычислений определите, равно ли среднее значение cos2 (t) cos2 (t) по [0, π] [0, π] 1 / 2,1 / 2.

      128.

      Объясните, почему графики квадратичной функции (параболы) p (x) p (x) и линейной функции ℓ (x) ℓ (x) могут пересекаться не более чем в двух точках. Предположим, что p (a) = ℓ (a) p (a) = ℓ (a) и p (b) = ℓ (b), p (b) = ℓ (b), и что ∫abp (t) dt> Abℓ (t) dt.∫abp (t) dt> ∫abℓ (t) dt. Объясните, почему ∫cdp (t)> ∫cdℓ (t) dt∫cdp (t)> ∫cdℓ (t) dt, когда a≤c 129.

      Предположим, что парабола p (x) = ax2 + bx + cp (x) = ax2 + bx + c открывается вниз (a <0) (a <0) и имеет вершину y = −b2a> 0. y = – b2a> 0. Для какого интервала [A, B] [A, B] ∫AB (ax2 + bx + c) dx∫AB (ax2 + bx + c) dx максимально велико?

      130.

      Предположим, что [a, b] [a, b] можно разделить на подинтервалы a = a0

      1. Объясните, почему ∫abf (t) dt = A1 + A2 + ⋯ + AN. Abf (t) dt = A1 + A2 + ⋯ + AN.
      2. Затем объясните, почему | ∫abf (t) dt | ≤∫ab | f (t) | dt. | ∫abf (t) dt | ≤∫ab | f (t) | dt.
      131.

      Предположим, что f и g – непрерывные функции, такие что ∫cdf (t) dt≤∫cdg (t) dt∫cdf (t) dt≤∫cdg (t) dt для каждого подынтервала [c, d] [c , d] из [a, b]. [a, b]. Объясните, почему f (x) ≤g (x) f (x) ≤g (x) для всех значений x .

      132.

      Предположим, что среднее значение f по [a, b] [a, b] равно 1, а среднее значение f по [b, c] [b, c] равно 1, где a f по [a, c] [a, c] также равно 1.

      133.

      Предположим, что [a, b] [a, b] можно разбить. взяв a = a0 f на каждом подынтервале [ai − 1, ai] = 1 [ai − 1, ai] = 1 равно 1 для каждого i = 1,…, Ni = 1,…, N. Объясните, почему среднее значение f по [a, b] [a, b] также равно 1.

      134.

      Предположим, что для каждого i такого, что 1≤i≤N1≤i≤N, имеет место ∫i − 1if (t) dt = i.∫i − 1if (t) dt = i. Покажем, что ∫0Nf (t) dt = N (N + 1) 2. 0Nf (t) dt = N (N + 1) 2.

      135.

      Предположим, что для каждого i такого, что 1≤i≤N1≤i≤N, имеет место ∫i − 1if (t) dt = i2.∫i − 1if (t) dt = i2. Покажем, что ∫0Nf (t) dt = N (N + 1) (2N + 1) 6.∫0Nf (t) dt = N (N + 1) (2N + 1) 6.

      136.

      [T] Вычислите левую и правую суммы Римана L 10 и R 10 и их среднее значение L10 + R102L10 + R102 для f (t) = t2f (t) = t2 за [0, 1]. [0,1]. Учитывая, что ∫01t2dt = 0,33–, ∫01t2dt = 0,33–, до скольки десятичных знаков точность L10 + R102L10 + R102?

      137.

      [T] Вычислите левую и правую суммы Римана, L 10 и R 10 и их среднее значение L10 + R102L10 + R102 для f (t) = (4 − t2) f (t ) = (4 − t2) над [1,2]. [1,2]. Учитывая, что ∫12 (4 − t2) dt = 1.66–, ∫12 (4 − t2) dt = 1.66–, до скольки десятичных знаков точность L10 + R102L10 + R102?

      138.

      Если ∫151 + t4dt = 41.7133 …, ∫151 + t4dt = 41.7133 …, что такое ∫151 + u4du? ∫151 + u4du?

      139.

      Оцените ∫01tdt∫01tdt, используя суммы левой и правой конечных точек, каждая с одним прямоугольником.Как соотносится среднее этих сумм левой и правой конечных точек с фактическим значением ∫01tdt? ∫01tdt?

      140.

      Оцените ∫01tdt∫01tdt путем сравнения с площадью одиночного прямоугольника с высотой, равной значению t в средней точке t = 12.t = 12. Как эта средняя оценка соотносится с фактическим значением ∫01tdt? ∫01tdt?

      141.

      Из графика sin (2πx) sin (2πx) показано:

      1. Объясните, почему ∫01sin (2πt) dt = 0. 01sin (2πt) dt = 0.
      2. Объясните, почему в общем случае ∫aa + 1sin (2πt) dt = 0∫aa + 1sin (2πt) dt = 0 для любого значения a .

    Оставить комментарий